PNCB Acute Care questions

Pataasin ang iyong marka sa homework at exams ngayon gamit ang Quizwiz!

An echocardiogram for a child who underwent patch closure of a ventricular septal defect (VSD) demonstrates a small residual VSD patch leak. Which of the following statements is the MOST appropriate education for the child and caregivers regarding this finding? A. "To reduce the risk of infective endocarditis, antibiotics need to be taken prior to dental care" B. "To reduce the risk of sudden cardiac death, participation in competitive sports is restricted" C. "A cardiac catheterization is neede

A. "To reduce the risk of infective endocarditis, antibiotics need to be taken prior to dental care" Over time, the majority of residual defects close spontaneously as endothelialization occurs around the patch. However, the one requirement for residual VSDs is that all patients with "repaired congenital heart disease (CHD) with residual defects at the site or adjacent to the site of a prosthetic patch or prosthetic device" should receive antibiotic prophylaxis prior to any dental procedure due to the risk of infective endocarditis. Further intervention or invasive assessment such as a cardiac catheterization is not required as long as the residual defect is small, there is no hemodynamic significance, and there is no threat or harm to the patient. Repaired or unrepaired VSDs are not commonly associated with sudden death during sports. At 3 to 6 months after repair, asymptomatic athletes with no or a small residual defect and no evidence of pulmonary hypertension, ventricular or atr

Which of the following infants has the HIGHEST risk for occult bacteremia? A. 1 month old with decreased capillary refill and temperature of 38.8° C (102° F) B. 2 month old with fever to 38.5° C (101.4° F) and fussiness 3 days after immunizations C. 4 month old with rhinorrhea, wheezing and a temperature of 39.2° C (102.6° F) D. 6 month old with fever of 39.4° C (103° F) whose siblings recently had diarrhea

A. 1 month old with decreased capillary refill and temperature of 38.8° C (102° F) When an infant with fever presents to primary or acute care, the evaluation and management must be individualized based on age and risks for occult bacteremia. Children at higher risk for bacteremia include infants who are less than 1 month old or between 1 and 3 months of age, who appear toxic with no other obvious symptoms to suggest illness. As infants get older and receive immunizations, the evaluation of fever and the need for intervention can be modified based on presenting condition and neurological exam, along with symptoms such as vomiting, diarrhea, cough, or congestion.

Which of the following infants receiving mechanical ventilation is demonstrating readiness for extubation? A. An infant with bronchiolitis on pressure support ventilation with TV on 6-7 ml/kg B. An infant with chronic lung disease on volume ventilation with peak pressures of 30-34 cm H2O C. An infant who is post-op day 1 from AV canal repair with pulmonary HTN on volume ventilation with inhaled nitric oxide therapy D. An infant who is post-op day 3 from a congenital diaphragmatic hernia repair o

A. An infant with bronchiolitis on pressure support ventilation with TV on 6-7 ml/kg With pressure support ventilation the infant demonstrates the ability to initiate all breaths spontaneously & is determining the volume of these breaths on his/her own. Appropriate tidal volumes are 6-7 ml/kg. The infant with chronic lung disease has very high peak pressures, which is a marker of poor lung compliance and thus requires continued mechanical ventilation. Normal peak pressures in infants should be < 20 cm H2O. The infant who is post-operative from AV canal repair is receiving inhaled nitric oxide, which necessitates mechanical ventilation for delivery. Additionally, concern for pulmonary hypertension and other potential associated crises warrant postponing extubation until medical management of pulmonary hypertension is optimized. The infant who is postoperative from a congenital diaphragmatic hernia repair is currently requiring a high PEEP to maintain oxygenation and distention of the

Which of the following would be included in the anticipatory guidance provided to a family and their child who sustained a superficial, partial thickness second degree burn? A. Area will heal in 2-3 weeks B. Wound will need to be kept dry C. Hair in that area will no grow back D. Wound will require a skin graft to heal

A. Area will heal in 2-3 weeks First degree: Erythematous, painful, and dry Second degree burn: Erythematous, moist, and sensate, and are divided into superficial partial-thickness and deep partial-thickness wounds. Third degree/full thickness injury: Leathery, dry, and insensate. The importance of differentiating depth of injury relates to the fact that a superficial partial-thickness burn will re-epithelialize in approximately 2-3 weeks, whereas a deep partial-thickness heals by re-epithelialization and contracture. To avoid scarring, the deeper wound must be treated with skin grafting. Full-thickness wounds, except those that have a small surface area, should either be excised and closed primarily or grafted with the child's skin. The objective of wound care is to avoid infection and protect the wound from further injury. Systemic antimicrobial prophylaxis is not used, although topical antimicrobials are used. The wound should not be allowed to dry out. It should be covered with

Prior to a child with trisomy 21 participating in contact sports, an evaluation should be completed for which of the following musculoskeletal abnormalities? A. Atlantoaxial instability B. Hip subluxation C. Osteoporosis D. Patellar instability

A. Atlantoaxial instability Typical physical exam findings include hypotonia, broad nasal bridge, epicanthal folds, low-set ears, macroglossia, and simian creases. Cardiac defects, including endocardial cushion, septal defects, and tetralogy of Fallot. Extracardiac manifestations include gastrointestinal abnormalities such as duodenal atresia, tracheoesophageal fistula, and Hirschsprung disease. Musculoskeletal abnormalities associated with Down syndrome are typically attributable to collagen abnormalities, which results in increased laxity of ligaments connecting bones to muscle or to other bones. The major spinal abnormality being atlantoaxial instability.

A 13-month-old female with a history of developmental delay and hypotonia presents with new onset seizures, with the following measurements obtained during assessment: weight 18.7 lb (8.5 kg), length 28.3 inches (72 cm) - both in 25 %ile for age , head circumference 14.75 inches (40 cm). Her head circumference is below the 2 %ile After achieving control of seizures, which diagnostic testing should be performed FIRST? A. Brain MRI B. Head CT C. Skeletal survey D. Skull series

A. Brain MRI Imaging by MRI for microcephaly provides more structural information than CT without the radiation exposure, and identifies abnormalities including lissencephaly, pachygyria, and polymicrogyria. CT is most useful in evaluating for presence of calcifications, which are associated with congenital viral infections, Sturge-Weber syndrome, tuberous sclerosis, and intracranial lipoma in newborns, and with some vascular and neoplastic conditions in older children.

A toddler is being evaluated after failing to meet developmental milestones. Assessment reveals lack of protective reflexes, increased deep tendon reflexes, peripheral hypertonia, weakness, and cognitive impairment. The MOST likely diagnosis is: A. Cerebral palsy B. Guillain-Barre syndrome C. Muscular dystrophy D. Spinal muscular atrophy (SMA)

A. Cerebral palsy When a child fails to meet gross and/or fine motor development milestones, cerebral palsy (CP) is likely a differential diagnosis. It is a non-progressive condition; thus, the loss of previously acquired skills, developmental slowing or regression, or signs of a lower motor neurologic disorder would point towards another diagnosis, such as spinal muscular atrophy. Some children have cognitive impairments too. Muscular dystrophy commonly presents with a gait disturbance due to hip girdle weakness. Children are usually late to learn to walk, move slower than others, display toe walking, rolling gait, and a waddle. They may have a Gower sign and other language or developmental delays. Weakness is progressive, starting with voluntary muscles close to the hips and pelvis. SMA is a degenerative disease leading to muscle fiber atrophy and neuronal cell death. It frequently starts with the loss of previously attained skills and has a progressive decline in muscle strength. C

A toddler who has multisystem organ failure is mechanically ventilated and requires placement of a hemodialysis catheter. For the procedure, which neuromuscular blocking agent is the BEST choice? A. Cisatracurium (Nimbex) B. Pancuronium C. Succinylcholine (Anectine) D. Vecuronium

A. Cisatracurium (Nimbex) Cisatracurium: Onset 2-3 minutes, duration of action ~ 30-40 minutes It is FDA-approved for children 1 mo + Its onset & duration allow for timing of the dose to provide paralysis for a vascular access procedure, & lacks dependence on hepatic or renal function allows for its effective use without potential accumulation in the setting of multiorgan failure. Succinylcholine (Anectine) only depolarizing neuromuscular blocker used clinically. Rapid onset (~ 60 seconds) & very short duration of action (10 minutes or less). Its short duration of action makes it a less desirable choice for a vascular access procedure. Pancuronium: slower onset & longer duration of action. It would not be the best choice for a child with multisystem organ failure. Vecuronium: Steroid-based derivative of pancuronium is more lipid-soluble and is metabolized more in the liver than its predecessor. It also shares pancuronium's potential to accumulate in the setting of renal or hepatic dy

Which of the following findings would be MOST concerning for a child with Factor V Leiden deficiency? A. Femur fracture B. Hematochezia C. Periorbital petechiae D. Persistent epistaxis

A. Femur fracture (An inherited disorder that results from an alteration in the amino acid sequence of the factor V molecule, which makes it resistant to protein C and protein S, resulting in a prothrombotic state. While a femur fracture can cause major blood loss, the most concerning issue is the related trauma to the limb and vascular injury with the fracture. The combination of trauma and Factor V Leiden deficiency places the child at high risk for thrombosis and possible embolus)

When informing a family that a report is being made to Child Protective Services for suspected abuse, it is MOST important to A. Focus on the child's well being B. Identify the suspected perpetrator C. Identify legal requirements for mandatory reporting D. Share details with family to avoid misunderstanding

A. Focus on the child's well being

An adolescent who was previously overweight has reported a decline in her usual excellent school performance, muscle weakness, and weight loss. Laboratory screening for thyroid levels include a decreased TSH and elevated free T4. The MOST likely diagnosis is: A. Graves' disease B. hyperparathyroidism C. hypothyroidism D. iodine toxicity

A. Graves' disease Graves' disease is the most common cause of acquired hyperthyroidism. Common symptoms include the inability to concentrate, so a dramatic decline in school performance may be the first clinical sign. Nervousness, fatigue, weight loss, diarrhea, heat intolerance, palpitations, restless sleep, and exercise intolerance are other associated findings. Laboratory results associated with Grave's disease and hyperthyroidism include low TSH and elevated free T4 levels. Hypothyroidism usually does not result in acute or significant clinical findings, but over time children will develop a decline in growth velocity while still gaining weight. Children with hypothyroidism will have decreased levels of T4, but elevated TSH. Iodine toxicity results in hypothyroidism and hyperparathyroidism causes an increase* of circulating parathyroid hormone, which causes

A 3 day old with a lactate of 7 mg/dL is suspected of having an inborn error of metabolism. Concurrently with stabilization of cardiovascular and respiratory functions, which of the following interventions should be the FIRST priority? intravenous dextrose A. IV dextrose B. IV amino acids C. NG feeding of breast milk or formula D. NG administration of sodium benzoate

A. IV dextrose Inborn errors of metabolism result in the body's inability to properly break down proteins, carbohydrates, or fats. Stabilization of airway, breathing, and circulation is the first priority. Concurrently, all previous enteral intake should be stopped and intravenous dextrose and hydration begun. Until further diagnostic information is available, protein should be avoided. Sodium benzoate may be necessary as a later step for control of hyperammonemia.

Critical congenital heart disease (CCHD) screening is completed in a 1-day old neonate and subsequent hyperoxia test is performed with the following results: Measurements in RUE - CCHD screening 78%, hyperoxia test (100% oxygen) 79%. LLE - CCHD screening 76%, hyperoxia test 77% Physical exam reveals increased irritability and decreased bilateral femoral pulses. After obtaining vascular access, what is the next BEST course of action? A. Initiate alprostadil B. Initiate dopamine C. Obtain an echo

A. Initiate alprostadil An oxygen saturation (SpO2) greater than 95% in both extremities is a "passed screen" whereas an SpO2 less than 90% in the right hand or foot is a "failed screen" and requires further work up. A failed screen can be due to multiple reasons with the most critical being CHD. While an echocardiogram provides a definitive diagnosis, obtaining one can be time prohibitive for these critical neonates. It is important to note that once a concern for cyanotic heart disease is established, continued administration of supplemental oxygen should be avoided as it can further exacerbate the problem by reducing systemic blood flow. Ultimately, for these neonates, systemic circulation must be re-established and maintained. This occurs through re-opening of the patent ductus arteriosus (PDA) with alprostadil, a synthetic prostaglandin. Cardiac defects which rely on PDA blood flow to maintain systemic circulation include critical aortic stenosis, hypoplastic left heart syndro

An otherwise healthy infant, who is up to date on immunizations, has harsh, high-pitched inspiratory noises that have been present since birth and worsen with agitation. Which of the following is the MOST likely diagnosis? A. Laryngomalacia B. Posterior laryngeal cleft C. Tonsillar hypertrophy D. Vascular ring

A. Laryngomalacia Treatment is typically observation as most symptoms resolve spontaneously with growth. Associated reflux should be treated appropriately. For those with severe obstruction, surgical supraglottoplasty may be indicated. Clinical signs of tonsillar hypertrophy include possible OSA, snoring & mouth breathing, & symptoms that worsen during sleep. A posterior laryngeal cleft is due to a deficiency in the midline of the posterior larynx and causes aspiration and respiratory distress. In severe cases, the cleft extends so inferiorly that there is no separation between the trachea and esophagus, creating a tracheoesophageal fistula. A vascular ring is an anomaly of the aortic arch that causes compression of either or both the esophagus and trachea. Presenting symptoms include a classic "seal-bark" cough, respiratory distress, dysphagia, slow feeding, and recurrent respiratory infections

An adolescent who is critically ill is receiving a Propofol infusion and develops refractory bradycardia. Propofol infusion syndrome is suspected with which additional finding? A. Metabolic acidosis B. Worsening coagulopathy C. New onset seizure activity D. Abrupt onset of hyperthermia

A. Metabolic acidosis Propofol is a general anesthetic agent. It is used off-label in pediatric populations for deep procedural sedation. Propofol has a rapid onset of action and short duration of action, which requires repeated dosing or continuous infusion for maintenance of effects. Propofol carries an FDA warning (not a U.S. Boxed Warning) for a rare but serious and potentially fatal adverse effect termed "propofol infusion syndrome". Signs of propofol infusion syndrome include refractory bradycardia, metabolic acidosis, hyperkalemia, lipemia, rhabdomyolysis, hyperkalemia, myoglobinuria and renal failure. Propofol infusion syndrome is often associated with administration of propofol in higher doses, greater than 80 mcg/kg/min (or 5 mg/kg/hour), and continued use for longer than 48 hours, although it has been reported with infusions of shorter duration and lower rates.

Enteral nutrition is initiated in a child with severe malnutrition. Laboratory values indicative of refeeding syndrome include a magnesium level of 1.5 mg/dL, and: A. Phosphate 3.0 ml/dL & potassium 2.8 mEq/L B. Phosphate 3.0 ml/dL & potassium 6 mEq/L C. Phosphate 7.0 ml/dL & potassium 2.8 mEq/L D. Phosphate 7.0 ml/dL & potassium 6 mEq/L

A. Phosphate 3.0 ml/dL & potassium 2.8 mEq/L (Refeeding syndrome occurs when malnourished patients are refed too aggressively leading. It can occur within 1-3 days after reinstitution of nutrition. The major manifestations include fluid overload, hypophosphatemia (phosphate less than 3.5 mg/dL), hypokalemia (potassium less than 3.5 mEq/L), hypomagnesemia (magnesium less than 1.8 mg/dL), and thiamine deficiency. Complications include heart failure, dysrhythmias, respiratory muscle weakness, seizures,

In infants younger than 6 months of age diagnosed with pertussis, the early manifestations following the catarrhal stage MOST typically include gasping, bradycardia, and: A. apnea B. cough C. fever D. rash

A. apnea Pertussis in infants younger than 6 months can be atypical with a short catarrhal stage, followed by gagging, gasping, bradycardia, or apnea. Based on historical data, approximately 67% of infants with pertussis are likely to experience apnea. Young infants often do not cough (lacking the multitude of cough receptors that older children and adults have) even when severely affected with a respiratory illness. Fever is generally absent or minimal in patients of all ages.

Injuries MOST frequently occurring with abusive head trauma include retinal hemorrhages and: A. Rib fractures & classic metaphyseal lesions B. Classic metaphyseal lesions & bruising in an arched pattern C. Clavicular fractures & circumferential burns D. Rib fractures & bruising in an arched pattern

A. Rib fractures & classic metaphyseal lesions Injuries that are often associated with AHT include classic metaphyseal lesions (CML), rib fractures, and retinal hemorrhages. CML occur from traction or acceleration-deceleration forces and are highly suspicious for abuse. Rib fractures, especially posterior rib fractures, are a type of blunt trauma that are also highly specific for abuse. Various types of bruising patterns are associated with child maltreatment including bruising in an arched pattern, which is typical to bite marks. Bruising of any kind in a child under 4 months of age should raise concern for abuse, but not all bruising is associated with AHT. In children under 4 years of age, certain patterns of bruises or bruises located on the torso, neck, or ears are most concerning. Circumferential burns are associated with immersion into a hot liquid and typically have a clear line of demarcation. A multitude of fracture patterns are associated with abuse, although a clavicula

Following a motor vehicle collision, an adolescent presents with hemoptysis, tachypnea, and a pulse oximetry (SpO2) reading of 85% on a non-rebreather mask. Chest radiograph reveals right upper and middle lobe consolidations with subcutaneous emphysema. Following intubation, initial oxygenation index (OI) is 15. The MOST appropriate ventilator management strategy includes permissive hypercapnia and: A. TV of 6-8 ml/kg & PaO2 of 55-80 mmHg B. TV of 6-8 ml/kg & PaO2 of 70-95 mmHg C. TV of 9-10 ml

A. TV of 6-8 ml/kg & PaO2 of 55-80 mmHg Diagnosis is pulmonary contusion AMB hemoptysis, respiratory distress, and hypoxia, with subcutaneous emphysema and consolidation on CXR . Management includes targeting low tidal volumes (6-8mL/kg), permissive hypercapnia, and titration of PEEP and FiO2 to maintain lower oxygenation goals (PaO2 55-80 mmHg) to prevent oxygen toxicity.

For the past 4 days a previously healthy infant has had a fever and upper respiratory symptoms. Vital signs and laboratory values include T-38.7, HR 156, RR 32, BP 86/44, O2 92%, Na+ 132, K+ 4.5, glucose 100, ionized Ca+ 0.95, Phos 6.3, WBC 130,000, Hgb 6.1 & Plt 10,000. Which of the following is the PRIORITY intervention? A. administer a fluid bolus and consult oncology B. administer calcium gluconate and consult nephrology C. obtain blood cultures and administer IV ceftriaxone (Rocephin) D. a

A. administer a fluid bolus and consult oncology The combination of hyperleukocytosis, thrombocytopenia, and anemia should raise red flags for leukemia. It is imperative to involve the oncology service for treatment.

In an adolescent who has ingested a large amount of diphenhydramine (Benadryl), expected physical exam findings include: A. agitation, mydriasis, and flushing B. mydriasis, diaphoresis, and tachycardia C. bradycardia, hyperthermia, and flushing D. agitation, bradycardia, and hyperthermia

A. agitation, mydriasis, and flushing Anticholinergics act at muscarinic receptors of the central nervous system, targeting organs of the parasympathetic nervous system and the sympathetic nervous system. Anticholinergics include antihistamines, atropine, cyclic antidepressants, and phenothiazines, among other medications. The resulting symptoms at supratherapeutic levels will include impaired sweating, warm/dry skin, diminished intestinal peristalsis, tachycardia, mydriasis, urinary retention, flushing, hyperthermia, and hypertension. The mnemonic used to facilitate quick recognition of an anticholinergic toxidrome is "Mad as a Hatter; Blind as a Bat; Red as a Beet; Hot as a Hare; Dry as a Bone".

An adolescent with a large mediastinal mass is diagnosed with non-Hodgkin lymphoma. The MOST immediate concern is related to the risk for: A. airway compression B. cardiovascular collapse C. immunosuppression D. tumor lysis syndrome

A. airway compression Mediastinal masses commonly present with symptoms including dyspnea, orthopnea, stridor, and wheezing, which are symptoms representative of airway compression. The mass can be the result of a thrombus or enlarging soft tissue of malignant or non-malignant etiology. As the mass grows, or as hemodynamics change, the risk of acute airway compression increases. History commonly reveals that the child will not tolerate lying supine due to acute worsening of symptoms and will report sleeping on several pillows or in a chair for breathing comfort. In addition, until the mass responds to therapies and decreases in size, general anesthesia is avoided due to decreased airway tone. Cardiovascular instability can also occur in children with large mediastinal masses; however, it is generally not the most concerning complication. Immunosuppression and tumor lysis syndrome are risks of and complications following initiation of treatment but neither condition is the primary com

A 4-day-old neonate born at term presents with a one-day history of poor oral intake and increased irritability. Vital signs obtained while the infant is in a quiet state include HR 170, RR 80, BP 88/52 (65) taken in the right arm. Physical assessment reveals normal to bounding upper extremity pulses, weak femoral pulses, and liver border palpable 4 cm below the right costal margin. Which of the following is the PRIORITY medication to initiate? A. alprostadil B. epinephrine C. furosemide D. milr

A. alprostadil The neonate in this scenario has a ductal-dependent congenital heart lesion (coarctation of the aorta or hypoplastic left heart syndrome). A decrease in cardiac output due to physiologic closure of the ductus arteriosus leads to shock and eventual death that can be reversed only if the ductus arteriosus is reopened. Alprostadil administered as a continuous infusion can reopen the ductus by causing vasodilation and muscle relaxation through direct action on vascular and ductus arteriosus smooth muscles. Milrinone is routinely used in pediatric heart failure and post-cardiopulmonary bypass management. While milrinone may be recommended in this neonate as a second-line treatment, it is not the priority and should only be done with expert consultation. Furosemide is a diuretic and is not appropriate in the treatment of symptomatic ductal dependent lesions, despite the presence of symptoms of congestive heart failure such as tachypnea, tachycardia, and hepatomegaly. Epine

A 6 month old with profound hearing loss since birth develops seizures. The MOST likely differential diagnoses includes meningitis and: A. congenital CMV infection and genetic syndrome B. genetic syndrome and chronic otitis media C. congenital CMV infection and cholesteatoma D. chronic otitis media and cholesteatoma

A. congenital CMV infection and genetic syndrome The list of possible causes for hearing loss is extensive, but can be grouped by infectious, genetic or physical causes. Thirty percent of children with diagnosed congenital hearing loss have associated or causative syndromes; the most common infectious cause of congenital sensorineural hearing loss is cytomegalovirus (CMV) infection during fetal development. Any infant who presents with acute onset of seizure activity must have infectious etiologies ruled out, including meningitis.

A child is requiring a positive end expiratory pressure (PEEP) of 10 with mechanical ventilation. Which of the following hemodynamic effects is anticipated? A. decreased preload B. decreased afterload C. increased preload D. increased afterload

A. decreased preload Positive end expiratory pressure (PEEP) increases intrathoracic pressure proportionally such that the higher the PEEP, the higher the intrathoracic pressure. High intrathoracic pressure decreases pulmonary capillary pressure and in turn decreases venous return to the right heart. Decreased venous return to the right heart is synonymous with decreased preload. This effect can be dramatic and result in hypotension or other signs of inadequate perfusion. Increasing PEEP also decreases afterload; however, the hemodynamic effect of this is less consequential in most patients. Increasing PEEP does not increase preload or afterload. Mechanisms of increased preload primarily include increased fluid volume, increased sympathetic tone, or left heart failure. Mechanisms of increased afterload primarily include increased aortic pressure or increased systemic vascular resistance.

Which of the following medications has the potential for abuse and is popular among adolescents because of its euphoric effects? A. dextromethorphan B. diphenhydramine C. guaifenesin D. loratadine

A. dextromethorphan Dextromethorphan (DM) is an antitussive that is an ingredient in many non-prescription cough and cold preparations. Because it is a legal, non-controlled substance, it has been easy for adolescents to obtain and abuse. It is reported to have effects similar to barbiturates, LSD, and PCP. Chemically, DM is a relative of morphine, and if taken in large doses can cause mental status changes, euphoria, hallucinations, visual disturbances, tachycardia, hypertension, slurred speech, and ataxia. A compounding factor in DM abuse is that there is a plethora of information available on the internet regarding its abuse potential, as well as details about the amount needed to derive the desired effects. For these reasons, this medication has recently been restricted for purchase and is not available over the counter. Diphenhydramine is an antihistamine that in large doses causes a central nervous system depression, sedation and respiratory depression, but will not cause euphor

A school-age child presents with a fever for 5 days and nontender, reddened, flat lesions on soles and palms, and raised, tender lesions with pale centers on finger and toe pads. The MOST likely etiology is: A. endocarditis B. tick-borne illness C. neuroblastoma D. osteomyelitis

A. endocarditis Janeway lesions are nontender erythematous macules presenting on the soles and palms. Osler nodes are tender subcutaneous violaceous nodules mostly on the pads of the fingers and toes. Both of these phenomena, if present, are highly suggestive of endocarditis. The clinical manifestations of endocarditis are variable and can present insidiously or catastrophically, depending on various factors. Common symptoms include fever, malaise, and weight loss. A cardiac murmur and tachycardia may be noted on exam. Osteomyelitis is an infection involving bone. Neuroblastoma is a tumor arising from sympathetic ganglion cells of the nervous system. Tick-borne illnesses, including Lyme disease, Rocky Mountain Spotted Fever (RMSF), and tularemia, commonly present with constitutional symptoms including fever, headaches, and arthralgias. Rashes can occur, such as the characteristic erythema migrans in Lyme disease or a petechial rash with RMSF, but not the findings noted in this scen

An adolescent with bulimia nervosa who engages in vomiting and laxative and diuretic use is at risk for which metabolic abnormalities? A. hypochloremia, hypokalemia, metabolic alkalosis B. hypochloremia, hyperkalemia, metabolic acidosis C. hyperchloremia, hypokalemia, metabolic alkalosis D. hyperchloremia, hyperkalemia, metabolic acidosis

A. hypochloremia, hypokalemia, metabolic alkalosis Abnormalities in electrolytes can be present in anorexia nervosa, bulimia nervosa, and during refeeding. Taking in too much or too little fluids can cause metabolic derangement, and both the vomiting and laxative or diuretic use in bulimia nervosa can lead to hypochloremia, hypokalemia, and metabolic alkalosis. Obtaining laboratory studies including electrolytes is an important part of the initial assessment when an eating disorder is suspected.

The main objective of parenteral nutrition for a child with septic shock who is mechanically ventilated and receiving inotropic support is to maintain: A. positive nitrogen balance B. adequate hydration C. metabolic alkalosis D. GI barrier function

A. positive nitrogen balance Children who experience acute systemic illness are prone to malnutrition as a result of hormonal and metabolic changes associated with the systemic inflammatory response triggered by the condition. Such responses include changes in glucose metabolism and increased protein turnover and breakdown, which result in increased energy expenditure and a negative nitrogen balance. A positive nitrogen balance supports tissue repair and healing. The objective of nutrition in illness is to manage stress response preventing hypermetabolism, so maintenance of a normal pH or bicarbonate level is ideal. Even though nutrition does affect gut barrier functions, this is not the main objective of total parenteral nutrition. In the case of shock, despite the need to maintain circulating volume, adequate hydration is not the primary objective of intravenous nutrition.

Which of the following findings in a preschooler with ear pain and facial nerve palsy is MOST significant for possible mastoiditis? A. postauricular swelling B. postauricular lymphadenopathy C. pain with movement of the neck D. pain with movement of the tragus

A. postauricular swelling Mastoiditis is a complication of acute or chronic otitis media involving infection of the mastoid bone with its numerous air cells adjacent to the inner ear apparatus. Children with mastoiditis have findings of otitis media plus tenderness and swelling of the mastoid process. Any exam of the ear should assess for postauricular swelling or pain to exclude mastoiditis. Parotitis and postauricular lymphadenitis are also characterized by a tender mass behind the ear pinna. Lymphadenitis is separate from the mastoid process with palpable lymph nodes. Parotitis can extend affecting the preauricular region anterior to the margin of the mandible. Mastoiditis is always accompanied by findings of acute otitis media. Children with mastoiditis may have pain in the occipital region; if meningitis cannot be excluded by exam alone, then a lumbar puncture may be necessary.

A previously healthy school-age child presents with a progressive 2-month history of weakness and malaise, which has worsened recently. History is notable for recent vomiting and weight loss and physical examination reveals orthostatic hypotension. Notable laboratory results reveal hypoglycemia, hyponatremia, and hyperkalemia. Additional diagnostic testing should include: A. serum cortisol B. serum magnesium C. serum phosphorus D. urine sodium

A. serum cortisol A progressive 2-month history of weakness, malaise, vomiting, weight loss, and orthostatic hypotension are concerning for a variety of etiologies in this school-age child, including gastrointestinal, neuromuscular, and cardiac diseases, as well as fluid and electrolytes derangements. Laboratory findings including hypoglycemia, hyponatremia, and hyperkalemia, which represent a triad of laboratory abnormalities, are a hallmark for adrenal insufficiency. Adrenal insufficiency often presents insidiously in older children, fitting with the 2-month history of progressive symptoms. When concerned for adrenal insufficiency, a serum cortisol should be obtained to confirm the diagnosis and gauge severity. Other serum or urine electrolytes, such as sodium, phosphorus, or magnesium, are of limited utility in making a diagnosis in this scenario. It is possible that these electrolytes will also be abnormal; however, they will not lead the provider closer to a definitive diagnosis

A school-age child presents with severe abdominal pain and a palpable purpuric rash to bilateral lower extremities. Which question is MOST helpful in determining a diagnosis? A. "What has your child eaten today?" B. "Has your child complained of joint pain?" C. "Can you tell me who else lives in your home?" D. "Has your child been prescribed any medications recently?"

B. "Has your child complained of joint pain?" Henoch-Schönlein purpura (HSP) is an acute vasculitis that is the most common cause of purpura in children without thrombocytopenia. Abdominal pain, purpura, and arthritis are the classic triad. A thorough review of systems can illuminate the constellation of symptoms seen with this diagnosis and help to avoid unnecessary surgery. Another important sign that should be investigated in the review of systems and by exam is edema in dependent or distensible areas. Skin, bowel, and kidneys are the most commonly affected organs. Laboratory evaluation should include urinalysis to evaluate for occult hematuria and proteinuria as renal involvement is common and can become chronic in a small percentage of patients. Occasionally, intestinal complications may become severe and can include bowel perforation and intussusception. It is postulated that purpura on the bowel (submucosal edema and hemorrhage) acts as a lead point for intussusception.The ex

A preschooler presents with a two-day history of decreased urine output and diarrhea which has become bloody. Physical exam reveals a listless child with mild hypertension and without fever. In obtaining the history, the MOST helpful question is: A. "Has the child had any fevers in the past week?" B. "What has the child eaten in the past week?" C. "Does your child attend daycare?" D. "Has the child had any vomiting?"

B. "What has the child eaten in the past week?" Hemolytic uremic syndrome (HUS) is most common in preschool and school-age children but can occur in adolescents and adults. Shiga toxin-producing E. coli is the major cause of the diarrhea-associated illness, and can be transmitted via raw or undercooked beef contaminated with E. coli 0157:H7, as well as from lettuce, ice, apple cider, uncooked vegetables, contaminated swimming pools, lakes, water parks, and from person-to-person transmission. Children present with symptoms of watery diarrhea which has occurred for several days and usually changes to bloody diarrhea. The prodromal gastroenteritis can also include fever, vomiting, and abdominal pain. HUS is more obvious with the sudden onset of pallor, irritability, lethargy, and weakness. HUS can cause renal failure with laboratory findings of anemia, thrombocytopenia, hyponatremia, hyperkalemia, and increased BUN and creatinine. In obtaining a history, it is very important to identify

A child has recently undergone nephrectomy for resection of a Wilms tumor. Education for the family should be related to which NEXT expected treatment? A. Biologic therapy B. Chemotherapy C. Immunotherapy D. Radiation therapy

B. Chemotherapy Chemotherapy is always used in the treatment of a Wilms tumor. In pediatrics, radiation therapy is primarily used in the treatment of solid tumors. For Wilms tumors stages 3-5, radiation may be indicated in addition to resection and chemotherapy, but it is not always part of the treatment for stages 1-2. Biologic therapies currently in clinical use for pediatric patients include differentiation therapy, immunotherapy, small-molecule kinase inhibitors, and monoclonal antibody therapies. Differentiation therapy has been used in acute promyelocytic leukemia and neuroblastoma. An example of immunotherapy is stem-cell transplantation. Monoclonal antibody therapy is typically used in the context of bone marrow transplantation.

An infant receiving long term parenteral nutrition for a significant bowel resection is MOST at risk for which complication? A. Acute renal failure B. Cholestatic liver disease C. Dehydration D. Refeeding syndrome

B. Cholestatic liver disease Cholestasis is a common complication of parenteral nutrition. Decreased enteral intake leads to decreased stimulation and emptying of the gallbladder, causing an accumulation of substances in the hepatocytes and bile ducts that are normally excreted in bile (e.g., bile acids, bile salts, and cholesterol). Cholestasis is characterized by conjugated hyperbilirubinemia and jaundice. It can lead to fibrosis and may result in liver failure. With the correct calculation of maintenance fluid needs, proper ordering of PN should not lead to dehydration. Refeeding syndrome is a risk with the reinitiation of parenteral or enteral nutrition in a child who is malnourished. It is characterized by electrolyte disturbances and fluid overload. Gradual advancement of caloric intake from carbohydrate, and close monitoring of electrolytes with repletion as needed, helps prevent refeeding syndrome.

A school-age child with asthma presents with tachypnea and moderate retractions. Physical exam reveals a pulse oximeter of 89% in room air with loud, bilateral wheezes auscultated throughout exhalation. Initial treatment includes supplemental oxygen, inhaled short-acting beta-2-agonist (SABA) every 20 minutes for 1 hour, and oral systemic corticosteroids. Reassessment after initial treatment reveals no clinical change and peak expiratory flow (PEF) reading is less than 40% of expected. The most

B. Continuous inhaled SABA with ipratropium This child is having a moderate asthma exacerbation. Initial treatment includes supplemental oxygen to maintain oxygen saturations greater than 90%, inhaled short- acting beta2-agonist (SABA) by either nebulizer or metered-dose inhaler, administering up to 3 doses in the first hour, as well as systemic corticosteroids dosed every 12 hours. On reassessment, this child is exhibiting symptoms of a severe exacerbation, which include hypoxia, peak expiratory flow less than 40% with worsening or unchanged respiratory effort. Further treatment would include supplemental oxygen, nebulized SABA with ipratropium (hourly or continuous), systemic corticosteroids, and consideration of adjunct therapies. Criteria for intubation and mechanical ventilation include severe respiratory failure evidenced by hypoxia unresponsive to supplemental oxygen, persistent or increasing hypercapnia on blood gas analysis, worsening of mental status, and the absence of bre

A toddler presents with an initial episode of nephrotic syndrome. FIRST-LINE treatment is: A. Bicarbonate replacement B. Corticosteroids C. Phosphate binders D. Sodium supplementation

B. Corticosteroids Nephrotic syndrome (NS) is characterized by edema, proteinuria, hypoalbuminemia, & hyperlipidemia. The most frequent initial sign is periorbital edema, which is more prominent in the morning and decreases during the day. Other signs and symptoms include irritability, anorexia, abdominal pain, and diarrhea. Proteinuria is rapidly identified on urine dipstick. However, the preferred diagnostic study is a 24-hour urine collection for protein; findings of at least 40 mg/m2/hour or 50 mg/kg/day in young children or 4 grams/day in older children or adolescents is consistent with NS. The first-line treatment is high-dose corticosteroids. Either prednisone or prednisolone is dosed at 2 mg/kg/day or 60 mg/m2/day with a maximum daily dose of 80 mg as initial therapy. Initial therapy is continued until remission is achieved, which usually occurs within 4 - 6 weeks. Once remission is achieved, steroids are tapered off.

The MOST common viral infection following pediatric lung transplantation is: A. Adenovirus B. Cytomegalovirus (CMV) C. Epstein-Barr virus (EBV) D. Human herpesvirus 6

B. Cytomegalovirus (CMV) Respiratory viral infections in the post-lung transplant population can be mild to life-threatening. Etiologic agents include respiratory syncytial virus, influenza, human herpes viruses, Epstein-Barr virus, adenovirus and cytomegalovirus (CMV). CMV is the most common cause of serious viral infection following lung transplant. CMV can cause bronchiolitis obliterans, which can cause significant graft dysfunction and death. Those at highest risk for CMV pneumonia are seronegative recipients of lungs from seropositive donors.

The most appropriate INITIAL maintenance fluid for a child with pneumonia and adequate urine output who is not tolerating oral intake is A. D5 ½ normal saline plus potassium chloride 10 mEq/L B. D5 normal saline plus potassium chloride 20 mEq/L C. ½ normal saline plus potassium chloride 10 mEq/L D. normal saline plus potassium chloride 20 mEq/L

B. D5 normal saline plus potassium chloride 20 mEq/L

The administration of succinylcholine is CONTRAINDICATED for a male toddler with a family history of: A. Diabetes mellitus B. Duchenne muscular dystrophy C. Sudden cardiac death D. Tachyarrhythmia

B. Duchenne muscular dystrophy Rhabdomyolysis may occur when succinylcholine is administered to a patient with an undiagnosed myopathy, leading to life-threatening hyperkalemia. Males, especially toddlers, with a family history of Duchenne's muscular dystrophy may be yet undiagnosed; thus, succinylcholine administration should be avoided. Diabetes mellitus has an antagonizing effect when cisatracurium (Nimbex) is used, and the diagnosis itself is not a strict contraindication to the use of succinylcholine. Pancuronium can cause tachycardia and should be used with caution in patients at risk for tachyarrhythmias. A family history of sudden cardiac death should prompt the provider to investigate possible causes (e.g., long QT syndrome, cardiomyopathy) and avoid medications contraindicated with those conditions.

A child presents after a fall from a high ladder, awake and interactive with an initial Glasgow Coma Score (GCS) of 15. However, shortly after admission, the child's GCS acutely declines to 8. Which is the MOST likely diagnosis? A. Diffuse axonal injury B. Epidural hematoma C. Intraventricular hemorrhage D. Subdural hematoma

B. Epidural hematoma (The typical presentation includes an initial lucid interval but then rapid deterioration occurs as the size of the intracranial hemorrhage enlarges. Headaches with nuchal rigidity & vomiting can also be seen. Diffuse axonal injury usually occurs with motor vehicle collision in which there are rapid acceleration or deceleration forces causing shear trauma. Subdural hematomas are most commonly seen in younger children, especially those who suffer non-accidental trauma. Intraventricular hemorrhage (IVH) is seen in premature infants related to spontaneous bleeding of immature and fragile blood vessels in the subependymal germinal matrix of the premature brain. Presentation is often nonspecific and subtle, with IVH not typically seen after trauma.)

An interprofessional team is developing an algorithm to guide the ordering of diagnostic tests for children seen in their sub-specialty clinic. The team will review and grade relevant literature and determine studies applicable to their clinical setting. This approach promotes: A. Clinical research B. Evidence-based practice C. Quality improvement D. Work models

B. Evidence-based practice Evidence-based practice is the process of reviewing and analyzing the current evidence on a topic by searching published literature. Using standardized scoring, the literature is reviewed to identify the strength of the evidence based on rigor and study methodology. Once the available evidence is analyzed, the applicability to the specific clinical setting is considered to develop new practice guidelines.

A previously healthy toddler has respiratory distress that began with a cough yesterday during playtime followed by increased work of breathing. Focal exam reveals child to be alert, slightly tachypneic, with mild suprasternal retractions, and unilateral wheezing on auscultation. Which of the following diagnoses is HIGHEST on the differential? A. Asthma exacerbation B. Foreign body aspiration C. Pneumonia D. Tracheitis

B. Foreign body aspiration Foreign-body aspiration should be suspected when a previously healthy child experiences a sudden onset of wheezing. Although classic signs of foreign-body aspiration include choking or gagging, coughing, and asymmetrical breath sounds on auscultation, presentation can vary widely. Clinical manifestations range from acute respiratory distress to a child that is asymptomatic, depending on the location and characteristics of the foreign body as well as the duration of time that has elapsed. A careful history should be obtained. The age of the child in the scenario and the fact that the onset occurred during playtime are risk factors for foreign-body aspiration. The suddenness of onset is a key difference in presentation compared to the other potential etiologies.

An adolescent presents after a recent viral illness with numbness and a feeling of heaviness and weakness in bilateral lower extremities. The MOST likely diagnosis is: A. acute flaccid myelitis B. Guillain-Barré syndrome C. myasthenia gravis D. tick paralysis

B. Guillain-Barré syndrome GBS is an acute inflammatory demyelinating polyradiculoneuropathy. It typically presents post infectious illness with the typical timing of presentation about 10 days following the infection. The classic symptoms are areflexia, flaccidity, and symmetrical ascending weakness. Progression can vary in nature from rapid onset with progression to slower development over weeks. Typical symptoms start with numbness or paresthesia in the hands and feet, with a sensation of heaviness in the legs. The weakness is ascending and involves the arms, trunk, and bulbar muscles, and is often symmetric. The manifestation of AFM typically occurs within 7 days of a respiratory or febrile illness, and the resultant limb weakness may be to one or all four limbs. While AFM can be challenging to differentiate from GBS, weakness is more commonly asymmetrical and is most often seen in young children. Myasthenia gravis is an autoimmune condition where antibodies block the acetylch

A toddler presents with fever, abdominal pain, watery, non-bloody diarrhea, and decreased urine output that is dark amber and blood-tinged. Physical exam reveals petechiae and laboratory results include Hgb 8 gm/dl, RBC morphology +schistocytes, Plt 34,000, aPTT 30 secs, PT 10 secs & fibrinogen 250 mg/dl. Which diagnosis is the MOST likely? A. Disseminated intravascular coagulation (DIC) B. Hemolytic uremic syndrome (HUS) C. Henoch-Schonlein purpura (HSP) D. Thrombotic thrombocytopenic purpura

B. Hemolytic uremic syndrome (HUS) HUS is characterized by hemolytic anemia, thrombocytopenia, and acute kidney failure. The clinical presentation includes fever, abdominal pain, diarrhea (initially watery, non-bloody diarrhea which may progress to hemorrhagic colitis), pallor, jaundice, petechiae, hematuria, hematemesis, and oliguria or anuria. Diagnostic findings include elevated BUN, creatinine, and bilirubin, decreased hemoglobin with evidence of schistocytes on red blood cell smear, decreased platelet count, and normal coagulation labs. Thrombotic thrombocytopenic purpura involves thrombocytopenia, microangiopathic hemolytic anemia, kidney abnormalities, neurologic changes, and fever. DIC often presents with diffuse bleeding with prolonged clotting times and is typically caused by infection, trauma, malignancies, toxic or immunologic reactions, pregnancy complications, vascular irregularities, or liver failure. Children with Henoch-Schönlein purpura often experience a precedin

When assessing a toddler for failure to thrive, which of the following is MOST important in the INITIAL evaluation? A. BMI B. History C. Physical examination D. Serum albumin

B. History This should include dietary intake, feeding behaviors, current & past medical history, and psychosocial history. A diagnosis of FTT should not be made with a single set of anthropometric measurements, but with a series of measurements which allows the practitioner to evaluate if the child has fallen off the anticipated growth curve suddenly or if growth has been consistently suboptimal. Physical assessment is an important part of determining nutritional status, but may not yield as many findings as the history. While lab studies such as serum protein (albumin, prealbumin) may eventually be needed to further assess nutritional status, the history remains the most important investigative element. Additionally, the history and physical examination will assist in developing differential diagnoses that can guide further laboratory testing for a more targeted diagnostic work up.

An adolescent male presents with a history of occasional chest pain, shortness of breath, and dizziness during exercise. His ECG is abnormal with evidence of left ventricular hypertrophy and ST-segment changes. Family history reveals a grandfather who died of unknown etiology at 35 years of age. Which of the following diagnoses on the differential list is of MOST concern? A. First degree heart block B. Hypertrophic cardiomyopathy C. Myocarditis D. Wolff-Parkinson-White syndrome

B. Hypertrophic cardiomyopathy (HCM) HCM is a relatively common and potentially life-threatening form of cardiomyopathy with multiple causes. Approximately half of cases are inherited in an autosomal dominant pattern. The key screening point for risk of cardiomyopathy in a previously healthy person is a detailed family history since inheritance of mutations occurs in an autosomal dominant pattern. In a child or adolescent presenting with the above signs and/or symptoms, a direct relative with early, sudden, unexplained death indicates immediate concern for cardiomyopathy with further work up. Myocarditis is characterized by acute or chronic inflammation of the myocardium due to infectious, connective tissue, granulomatous, toxic or idiopathic causes, with viral infections considered to be the most common etiology. Wolff-Parkinson-White syndrome may also cause palpitations, dizziness, shortness of breath, and syncope. Acquired first-degree heart block may occur in otherwise healthy

A 2 day old with dysmorphic features is diagnosed with an interrupted aortic arch. Evaluation for which of the following is indicated? A. Coagulation defects B. Immune dysfunction C. Intestinal atresia D. Pulmonary sequestration

B. Immune dysfunction DiGeorge syndrome is associated with deletions in the 22Q11.2 chromosome. The phenotype typically includes cardiac outflow tract abnormalities, dysmorphic features, cleft palate, and aplasia or hypoplasia of the thymus and parathyroid. The cardiac abnormality most strongly associated with DiGeorge syndrome is an interrupted aortic arch. Thymic aplasia or hypoplasia may produce immune dysfunction that requires the aggressive management of infections. Administration of live vaccines is contraindicated. Seizures secondary to hypocalcemia may occur in the newborn period and typically resolve in the first year. Dysmorphic features associated with intestinal atresia occur most commonly in trisomy 21. Dysmorphic features occur in conjunction with coagulopathy in genetic disorders of the metabolic process of glycosylation. Pulmonary sequestrations are most often intrapulmonary lesions, but may also be extrapulmonary.

Which method provides the BEST estimate of energy requirements for enteral feeding of a school-age child who is mechanically ventilated for respiratory failure? A. Holiday Segar Method B. Indirect Calorimetry Measurement C. Schofield Equation D. World Health Organization Equation

B. Indirect Calorimetry Measurement Indirect calorimetry most accurately predicts energy expenditure through measuring oxygen consumption and carbon dioxide production, which is then used to calculate resting energy via the Weir equation. Measurements are taken via a continuous metabolic monitor and 15-minute measurements can provide 24-hour energy expenditure. Contraindications to indirect calorimetry include hemodialysis, continuous renal replacement therapy, FiO2 >60%, or significant endotracheal tube air leak. The use of predictive equations such as the World Health Organization Equation and Schofield Equations are unreliable. The Holliday Segar Method is used to calculate maintenance fluid requirements, not energy requirements.

Which of the following children is MOST at risk for opiate withdrawal? A. Infant intubated for respiratory failure receiving Dexmedetomidine (Precedex) infusion for 5 days B. Infant intubated for respiratory failure receiving a morphine infusion for 6 days C. Adolescent post-posterior spinal fusion receiving hydromorphone via PCA for 3 days D. Adolescent post-cardiac surgery receiving PO oxycodone PRN for 2 days

B. Infant intubated for respiratory failure receiving a morphine infusion for 6 days (Opiate withdrawal should be suspected in any child who has received opioids for a minimum of 3 days. After 5 days, approximately 50% of children are expected to experience some withdrawal symptoms. When opiates are administered for >10 days, withdrawal should be expected in 100% of patients)

In which of the following scenarios is the risk of abuse MOST concerning for an infant who presents with lethargy, poor weight gain, and possible seizure activity? A. Mother is a single parent of 4 children who are each a year apart in age B. Infant with cleft lip & palate & has difficulty feeding with a special device C. Mother & infant are living with the maternal grandmother who provides childcare D. Infant with low birth weight who requires fortification of formula for catch-up growth

B. Infant with cleft lip & palate & has difficulty feeding with a special device

Which of the following medications would be appropriate to safely sedate a school-age child in hypotensive septic shock who requires immediate intubation? A. Etomidate B. Ketamine C. Propofol D. Versed

B. Ketamine When intubation is indicated for septic shock, patients are at risk of worsening hypotension from direct myocardial depression and vasodilator effects of induction agents. Propofol, thiopental, benzodiazepines, and inhalation agents all carry these risks. Ketamine is the agent of choice due to its ability to maintain cardiovascular stability. While etomidate can also maintain cardiovascular stability, a single dose has been seen to increase mortality in patients with septic shock, most likely due to its inhibition of corticosteroid biosynthesis, and therefore, the current septic shock guidelines do not support its use.

A child presents with lethargy and generalized malaise. Physical exam is significant for jugular venous distention and a liver edge palpable at 5 cm below the right costal margin. A chest radiograph demonstrates cardiomegaly, pulmonary edema, blunting of the right costophrenic angle, but with no mediastinal shift. Which diagnostic study should be obtained NEXT? A. Chest CT without contrast B. Lateral decubitus chest radiograph C. Oblique chest radiograph D. Spiral CT

B. Lateral decubitus chest radiograph Symptoms consistent with a pleural effusion. Simple effusion is best evaluated by a lateral decubitus radiograph on which the fluid will layer out along the chest wall and can be graded as small, moderate, or large depending on the width of the rim measured from the inner margin of the ribs to the outer margin of the lung. Ultrasonography is also useful in diagnosis and follow-up of effusion. Depending on the size, location, and amount of distress the child is exhibiting, chest tube placement or surgical referral may be indicated for drainage. Chest CT is not reliable in characterizing pleural fluid. Oblique chest radiograph is helpful for diagnosis of pneumothorax. Spiral CT should be used to diagnose pulmonary embolism.

An infant presents with irritability, poor feeding, and a heart rate of 290 beats per minute. Following stabilization, anticipatory guidance provided to the caregivers should include that the infant will A. Need lifelong medication B. Likely outgrow the problem by 1 year of age C. Require surgical referral for congenital heart disease D. Likely need an automated implantable cardioverter defibrillator

B. Likely outgrow the problem by 1 year of age SVT is the diagnosis. All infants who present with SVT should undergo evaluation for CHD to rule out structural anomalies as the underlying cause. Many infants tolerate SVT well. If sustained more than 6 hours, signs of congestive heart failure can develop with clinical manifestations of irritability, tachypnea, poor feeding, and pallor. Acute management includes vagal maneuvers, administration of adenosine given rapid IV push, cardioversion, especially if patient is rapidly deteriorating, overdrive pacing, and administration of other IV medications such as amiodarone, esmolol, and propranolol. Automated implantable cardioverter defibrillators are used in patients with ventricular arrhythmias or in congenital or acquired heart disease associated with a high risk of ventricular arrhythmias and risk of sudden death.

A 10 month old is evaluated for a cough, conjunctivitis, and a rash. Physical examination reveals coryza, greyish-white spots on the posterior buccal mucosa, a generalized, macular, erythematous rash, and no cervical lymphadenopathy or changes to the peripheral extremities are noted. The MOST LIKELY etiology is: A. Kawasaki disease B. Measles C. Parvovirus B19 D. Roseola

B. Measles Measles is a highly contagious viral illness which presents with cough, coryza, conjunctivitis, and greyish-white spots on the posterior buccal mucosa, known as Koplik spots. Kawasaki diseasepresents with 4-5 days of fever and at least 4 of the following 5 signs or symptoms: nonsuppurative conjunctivitis, mucous membrane changes, peripheral extremity changes, polymorphous rash, or cervical lymphadenopathy. The human parvovirus B19 frequently manifests with a distinct facial rash that appears as a "slapped cheek", contrasted by a paler area around the mouth. Also known as erythema infectiosum (EI) or fifth disease, it typically includes a pruritic rash that extends from the trunk, buttocks, and extremities with a lace-like appearance, which can fluctuate in intensity responding to environmental changes such as temperature. Roseola, caused by human herpesvirus-6 (HHV-6), includes clinical manifestations of an erythematous maculopapular rash that typically appears once the f

A school-age child reports, "I usually need my inhaler 3 or 4 times each week." This frequency of short-acting beta-agonist use is consistent with what severity of asthma? A. Intermittent B. Mild persistent C. Moderate persistent D. Severe persistent

B. Mild persistent Asthma severity can be stratified by the frequency of occurrence of the following factors: daytime symptoms, nighttime awakenings, short-acting beta-agonist (SABA)use, and interference with normal activities. Pulmonary function tests can corroborate historical information. SABA use twice weekly or less is consistent with intermittent asthma; while use more than 2 days per week (but not daily and not more than once per day) is consistent with mild persistent asthma. Daily use or use multiple times per day are classified as moderate and severe persistent, respectively.

A child presents in acute respiratory distress after being kicked in the chest by a horse. Vital signs include SpO2 85%, HR 160/min, and BP 70/30 mmHg. Oxygen by facemask is initiated and a chest radiograph is obtained, demonstrating a contralateral shift in the mediastinum with deviation of the trachea. The PRIORITY intervention is: A. NSB at 20 ml./kg B. Needle thoracostomy C. Chest tube insertion D. Intubation

B. Needle thoracostomy A pneumothorax can cause collapse of the ipsilateral lung, and can lead to mediastinal shift, hypoxia, and hypercapnia. A mediastinal shift may cause intrathoracic structures to impede cardiac output and can cause cardiovascular collapse .A tension pneumothorax is a life-threatening emergency. Findings are consistent with tracheal deviation to the contralateral side, hypotension, tachycardia, and cyanosis. Immediate relief of tension pneumothorax, which is the initial priority for hemodynamic stabilization, is achieved by needle decompression. Needle thoracostomy involves the perpendicular insertion of a needle in the 2nd ICS at the midclavicular line. This action will relieve intrathoracic pressure, thereby improving venous return. A chest tube will need to be inserted after the tension pneumothorax has been decompressed. However, due to the need for sedation, sterile draping and preparation, and for obtaining a chest drainage system, the insertion of a chest

Following initial stabilization of a child with approximately 50% TBSA burns, a consult with which service is MOST essential to initial treatment? A. Infectious disease B. Nutrition C. Rehabilitation D. Social work

B. Nutrition Aggressive nutritional support is hallmark in reducing the resting energy expenditure and thus preventing some complications. Enteral nutrition is much preferred over parenteral nutrition and should be initiated within 24 hours of burn injury. For these reasons, early and ongoing consultation with a nutritionist is of utmost importance. Consultation of rehabilitation services may include PT, OT, & then evaluation for eventual transfer to inpatient rehabilitation. Involvement of PT & OT begin during initial medical care following stabilization to begin ROM exercises. When the wounds have stable epidermal closure (usually within 2 weeks after grafting or primary wound healing), focus is turned to wound and scar management. While infection is a serious complication of burn injury, advances in burn care, early recognition, and topical agents have decreased the risk of infection. Consultation with infectious disease is only necessary when infection is present and thus are not

A toddler is experiencing respiratory distress and high-pitched whistling evident on inspiration that have occurred following two days of fever and a barky cough. The MOST likely viral etiology is: A. Influenza B. Parainfluenza C. Respiratory syncytial virus D. Rhinovirus

B. Parainfluenza Most consistent with croup (laryngotracheobronchitis or LTB). The barky cough & stridor, manifested as high-pitched whistling sound on inspiration, are symptoms consistent with inflammation of the larynx and subglottic area. The etiology of croup can vary; however, parainfluenza viruses are the predominant cause. Influenza usually presents with fever, body aches, fatigue, and a runny or stuffy nose. Cough may be present but is not the distinctive barky cough associated with croup. Rhinovirus causes symptoms of the common cold and has also been linked with AOM & sinusitis. RSV is a common, acute respiratory tract infection that typically starts with upper respiratory tract symptoms including nasal congestion, rhinorrhea, and cough, and progresses to the lower respiratory tract. Findings associated with this progression include crackles, wheezes, and an ongoing cough.

Routine preventive healthcare for an infant with sickle cell disease includes A. The MMR vaccine at 6 months of age B. Penicillin prophylaxis until 5 years of age C. Pneumococcal polysaccharide vaccine at 6 months of age D. Sulfamethoxazole-trimethoprim prophylaxis until 5 years of age

B. Penicillin prophylaxis until 5 years of age

In a child with myasthenia gravis, which of the following symptoms would be MOST indicative of a myasthenic crisis? A. Eye drooling B. Respiratory distress C. Excessive salivation D. Muscle fasciculation

B. Respiratory distress (Characterized by weakness and fatigue of skeletal muscle tissue that results from autoimmune destruction of acetylcholine receptors (AchR). A crisis event, or myasthenic crisis, is an acute exacerbation of the disease process that results in severe weakness from dysfunction of the neuromuscular junctions. It is characterized by respiratory failure due to weakness of the airway or respiratory muscles. A cholinergic crisis, is a severe weakness caused by overtreatment with cholinergic medications used to treat MG & crisis present with excessive salivation, excessive lacrimation, diarrhea, sweating, pupillary constriction, and muscle fasciculation.

Which of the following physical assessment findings is consistent with a diagnosis of coarctation of the aorta in an asymptomatic school-age child? A. Continuous murmur at the left upper sternal boarder & bounding pedal pulses B. Right upper extremity hypertension & weak femoral pulses C. Papilledema & severe headache D. Central cyanosis & hypoxia

B. Right upper extremity hypertension & weak femoral pulses Physical examination will demonstrate absent, weak, or delayed femoral arterial pulses, hypertension measured in the right arm, and hypotensive readings for blood pressure measured in the lower extremities. Murmurs associated with coarctation can include a systolic thrill, ejection click, and/or a soft systolic ejection murmur. Right arm hypertension secondary to coarctation is usually not significant enough to cause acute hypertensive changes including fundoscopic findings (papilledema) or headaches. Coarctation of the aorta does not cause intracardiac mixing of blood, therefore, hypoxia and cyanosis are not seen.

A toddler with a history of failure to thrive presents with a 2-day history of vomiting and diarrhea. Initial lab workup reveals a moderate metabolic acidosis with a normal anion gap, and mild hypokalemia. The MOST likely diagnosis is: A. Diabetic ketoacidosis B. Type I renal tubular acidosis C. Type IV renal tubular acidosis D. Salicylate ingestion

B. Type I renal tubular acidosis The presence of a normal anion gap distinguishes a metabolic acidosis resulting from diabetes (diabetic ketoacidosis) or a toxic ingestion in which the anion gap would be increased, from an acidosis related to dysfunction in the distal tubules of the kidney where acid excretion is impaired. Renal tubular acidosis (RTA) type 1 may occur as a result of an inherited or acquired defect in the distal renal tubule that affects the secretion of hydrogen ions, and is a common presentation in children with failure to thrive. RTA type IV is secondary to a deficiency of aldosterone and is associated with hyperkalemia. When evaluating children with metabolic acidosis, the presence of a normal or abnormal anion gap is useful in determining the etiology especially in moderate to severe cases. A normal anion gap is 3 to 11 mEq/L. The anion gap should be evaluated in the context of the child's other labs and clinical history.

For an infant with presumed severe sepsis, which of the following is considered one of the therapeutic endpoints to fluid resuscitation? A. Hgb of 8.9 mg/dl B. UOP of >1 ml/kg/hr C. Mixed venous saturation (SVO2) of 65% D. Liver palpable 3 cm below the right costal margin

B. UOP of >1 ml/kg/hr Initial therapeutic endpoints in pediatric shock resuscitation are: capillary refill of ≤2 seconds normal BP for age normal pulses with no differential between peripheral and central pulses warm extremities urine output >1 mL/kg/hr normal mental status. Mixed venous saturation of greater than 70% and a cardiac index between 3.3 and 6.0 L/min/m2 should be targeted thereafter. For hemoglobin levels, therapeutic endpoint would be levels of 10 g/dL are targeted. After stabilization and recovery from shock and hypoxemia, a lower target hemoglobin level of more than 7.0 g/dL can be considered reasonable. Hepatomegaly and/or rales may be a sign of hypervolemia, which may suggest the need to cease fluid resuscitation and transition to inotropic support should hypotension persist, although it is not considered a therapeutic endpoint.

A neonate born at 30 weeks gestational age has a prolonged prothrombin time (PT) and activated partial thromboplastin time (aPTT) with a normal fibrinogen. Which intervention would MOST likely confirm the diagnosis? A. administration of vitamin K enterally B. administration of vitamin K IV C. transfusion of cryoprecipitate D. transfusion of fresh frozen plasma (FFP)

B. administration of vitamin K IV Vitamin K deficiency occurs with a variety of transient defects in clotting. It is more frequent in low-birth-weight premature infants and in those infants who suffer anoxic events, sepsis, or liver disease. Lab abnormalities in vitamin K deficiency include increase prothrombin (PT) and partial thromboplastin times (PTT) with normal fibrinogen. The diagnosis of isolated vitamin K deficiency can be confirmed if administration of a therapeutic dose of vitamin K results in a decrease in the PT. This correction can be confirmed in as little as 30 minutes after IV administration. Vitamin K deficiency can also complicate other coagulopathies.

Which psychomotor activity is MOST closely related to a 2 year old's development? A. hops on one foot B. builds a tower of 6 - 7 blocks C. copies a crude circle D. holds a crayon with fingers

B. builds a tower of 6 - 7 blocks A two year old is capable of speaking in short sentences, building a tower of 6 - 7 blocks, kicking a ball, jumping off the floor with both feet, and turning pages of a book. A 30 month old can copy a crude circle, and a 3 year old holds a crayon with his fingers. The two year old can stand on either foot alone but is usually not capable of hopping on one foot.

During rapid sequence intubation, what is the BEST method for primary confirmation of endotracheal tube placement? A. auscultation B. capnography C. visualization of chest rise D. condensation in endotracheal

B. capnography

A school-aged child with a history of an anaphylactic reaction to vancomycin presents with pneumonia and severe respiratory distress. Which of the following antibiotics is the BEST treatment option to provide empiric coverage for community-acquired, methicillin-resistant Staphylococcus aureus? A. cefepime B. clindamycin C. levofloxacin D. nafcillin

B. clindamycin Resistant organisms cause pathophysiologic changes similar to that caused by susceptible organisms. Choosing appropriate empiric antibiotics requires consideration of local patterns of resistance as well as host factors. Resistance to first-line antibiotics can result in treatment delays, allowing disease to worsen before effective treatment is begun. Multiple factors have contributed to development of resistant organisms, including inappropriate use of antibiotics. Community-acquired methicillin-resistant Staphylococcus aureus (CA-MRSA) most commonly causes respiratory and skin and soft-tissue infections and is susceptible to non-beta lactam classes of antibiotics including clindamycin and trimethoprim-sulfamethoxazole. Vancomycin has been the standard therapy for MRSA in inpatient settings. Additional choices include clindamycin and linezolid, which are available in enteral formulations.

The BEST way to assess a legal guardian's comprehension of informed consent for a procedure is to A. ask what apprehensions he/she has about the procedure. B. have person explain in his/her own words what is going to be done. C. ask if he/she has questions, and if there are none, obtain signature for consent. D. have a social worker talk with the parent or guardian to confirm understanding.

B. have person explain in his/her own words what is going to be done.

An infant with unrepaired tetralogy of Fallot is admitted for emesis. Urgent surgical intervention is warranted for which of the following findings? A. feeding intolerance B. hypercyanotic spells C. polycythemia D. tachycardia

B. hypercyanotic spells Tetralogy of Fallot (TOF) is a structural lesion comprised of right ventricular outflow tract (RVOT) obstruction, ventricular septal defect (VSD), aorta overriding the ventricular septum, and right ventricular hypertrophy. Hypercyanotic spells, which occur as a result of sudden cyanosis and hypoxia, can be triggered by a variety of mechanisms including hypovolemia, tachycardia, or irritability. These spells are an indication for surgical repair. If the infant presents with possible intercurrent illness (vomiting in this example), then the care team should weigh the risks and benefits of immediate surgical intervention versus a period of inpatient observation prior to repair. The infant experiencing hypercyanotic spells should not be discharged from the inpatient setting prior to surgical repair. Feeding intolerance can be a sign of CHF that may present prior to surgical repair of an acyanotic defect, as these infants have minimal to no RVOT obstruction and in

Which of the following findings seen on echocardiogram is MOST consistent with Marfan syndrome? A. aortic stenosis B. mitral valve prolapse C. pulmonic stenosis D. ventricular septal defect

B. mitral valve prolapse Marfan syndrome is a genetic disorder caused by mutations in the FBN1 gene, which encodes the protein fibrillin-1. This mutation results in an increase in transforming growth factor beta (TGF-β), resulting in problems with connective tissues throughout the body. The mutation has considerable variability in expression, but defects primarily include the skeletal, cardiovascular, and ocular systems. Marfan syndrome predominantly affects males with phenotypic features including tall height, hyperflexibility, arachnodactyly, scoliosis, and pectus carinatum. The potentially life-threatening features of the disorder include valvular insufficiency and aortic root dilation. Mitral valve prolapse is the leading cause of morbidity and mortality in children with Marfan syndrome. Valvular insufficiency can lead to heart failure, pulmonary hypertension, and death. Clinical manifestations of mitral valve prolapse may include a blowing and high-pitched murmur, arrhythmias,

A school-age child with upper respiratory symptoms for several days presents with complaints of midsternal chest pain. Vital signs are T 38.4, HR 160, RR 30 & BP 98/60. Physical examination is notable for jugular venous distention, muffled heart sounds, and a friction rub on auscultation. Electrocardiogram demonstrates PR-segment depression. The MOST likely diagnosis is: A. myocarditis B. pericarditis C. dilated cardiomyopathy D. hypertrophic cardiomyopathy

B. pericarditis The most common symptoms of pericarditis are fever, tachycardia, tachypnea, and chest and abdominal pain. URI symptoms may be present 10 - 14 days prior to presentation, representing viral pericarditis. Physical findings include jugular venous distention, muffled heart sounds, hepatosplenomegaly, and a friction rub, often described as a high frequency murmur with a scratchy quality. Electrocardiogram (ECG) changes consistent with pericarditis include PR-segment depression, low voltage QRS complexes. Hypertrophic cardiomyopathy (HCM) is an inherited disorder of the cardiac muscle characterized by a hypertrophied, non-dilated left ventricle. In dilated cardiomyopathy (DCM), the left ventricle is dilated and has poor systolic function. This results in decreased shortening and ejection fractions, which lead to increased end-diastolic volume and pressure. DCM is most commonly idiopathic but may be secondary to myocarditis, coronary artery disease, or due to other etiologi

In a child with suspected sepsis, which of the following laboratory findings is MOST consistent with a transition from aerobic to anaerobic metabolism? A. WBC 23,000/mm3 B. serum lactate 6.2 mmol/L C. platelet count 110,000/mm3 D. c-reactive protein 15 mg/L

B. serum lactate 6.2 mmol/L Sepsis is a systemic inflammatory response to infection. This process leads to decreased tissue perfusion and thus, decreased oxygen and nutrient delivery. This results in cellular injury and a transition to anaerobic metabolism in an attempt to meet cellular demands. Lactate is a byproduct of cellular injury and anaerobic metabolism. An elevated serum lactate signifies that anaerobic metabolism is occurring. Neither thrombocytopenia nor leukocytosis indicate a transition to anaerobic metabolism but are potentially indicative of sepsis. C-reactive protein (CRP) is a non-specific marker of inflammation seen with bacterial infection; however, it is not indicative of the transition from aerobic to anaerobic metabolism.

Which of the following characterizes "privileging" for a new nurse practitioner (NP)? A. The State Board of Nursing grants the ability to practice after the NP meets state defined criteria for safe practice B. A hospital grants the NP the ability to practice in the institution after confirming education, certification & licensure C. A hospital permits the NP to perform certain procedures after successfully demonstrating competence as determined by the institution D. A certification board certif

C. A hospital permits the NP to perform certain procedures after successfully demonstrating competence as determined by the institution Privileging is the authorization to provide specific services & perform certain procedures within an institution. It is determined by the institution that hires the practitioner. Placement of arterial lines is an example of a privileging practice.

In an adolescent, which hemodynamic findings are consistent with cardiac tamponade? A. CVP 5 mmHg and BP 80/45 mmHg B. CVP 5 mmHg and BP 150/90 mmHg C. CVP 20 mmHg and BP 70/55 mmHg D. CVP 20 mmHg and BP 120/60 mmHg

C. CVP 20 mmHg and BP 70/55 mmHg This response occurs when the accumulating fluid in the pericardial sac compresses the right atrium and ventricle of the heart, leading to decreased venous return, filling, stroke volume, and cardiac output. Clinically, children may present with dyspnea and fatigue. On assessment, vital signs will demonstrate tachycardia, high central venous pressure (CVP), narrowed pulse pressure, and pulsus paradoxus. Physical assessment can demonstrate jugular venous distension, muffled heart sounds, and hepatomegaly. Cardiac tamponade is most common following cardiac surgery but can also occur with acute pericarditis, congestive heart failure, and some immunologic conditions including system lupus erythematous. For most children, cardiac tamponade occurs rapidly and is a medical emergency.

A CPNP-AC working for a general surgery service is being asked to offer telehealth visits as an option for postoperative follow-up. Before offering this to a family, it is MOST important to consider if the patient is: A. 18 years of age or younger. B. More than 14 post-operative days since surgical procedure. C. Connecting to the telehealth visit from across state lines. D. Participating in the telehealth visit having had at least one prior on-site visit.

C. Connecting to the telehealth visit from across state lines. When providing telemedicine services, it is necessary for the provider to consider where the patient is located. Although the provider may be located in his/her usual practice setting, the patient location is what determines state regulations for the visit. Age parameters do not differ for telehealth visits. A child who is stable enough at home is generally stable enough to be seen for an electronic visit. The amount of time since the surgery is not a likely determinant in how the child can be seen. While an on-site encounter is obviously required for surgery, post-operative evaluation may be feasibly offered through a telemedicine visit.

Manifestations of abdominal injuries secondary to a blast mechanism are typically: A. Acute & immediately apparent B. Observed less than 6 hours after exposure C. Delayed 8-36 hours after exposure D. Chronic & slow to develop

C. Delayed 8-36 hours after exposure (Blast attack, as seen in explosions, causes extensive compression and distortion of the gastrointestinal (GI) tract & other air-filled organs. Manifestations generally delayed, presenting 8-36 hours after exposure.)

A newborn female whose weight and length are between the 2nd and 10th percentile, respectively, for gestational age has significant pedal edema, a low hairline, and loose skinfolds at the nape of the neck. Which screening test is MOST appropriate? A. ACTH stimulation test B. Contrast enema C. Echocardiogram D. Head ultrasound

C. Echocardiogram The symptoms are characteristic of Turner syndrome. Many infants who have Turner syndrome exhibit a low birth weight and decreased length. Cardiac defects are commonly associated with Turner syndrome, and a comprehensive cardiovascular evaluation including echocardiography and consultation with a cardiologist specializing in congenital heart defects should be done. The most common cardiac defects occurring in females with Turner syndrome include bicuspid aortic valve, ascending aortic dilatation, coarctation of the aorta, and partial anomalous pulmonary venous return. Turner syndrome is not known to be associated with intracranial, adrenal, or gastrointestinal anomalies.

After an aortic arch repair, a child begins feeding and has milky-white drainage from the chest tube 24 hours later. Pleural fluid analysis demonstrates a triglyceride level of 200 mg/dL and 90% lymphocytes. Management for this condition should include a diet with: A. Low protein & low fat B. Low protein & high fat C. High protein & low fat D. High protein & high fat

C. High protein & low fat Chylothorax is caused by injury to the thoracic duct or associated lymphatic channels. Presentation can include respiratory distress or hypoxia. Most often seen is a "creamy" or turbid-appearing drainage from the post-operative chest tube. CXR may reveal a pleural effusion. Definitive diagnosis is made by evaluation of chest tube drainage for cell count and triglyceride level, confirmed with cell count revealing more than 80% lymphocytes (>1000 cells/microliter) and/or a triglyceride level greater than 110 mg/dL. The diagnosis can also be made retrospectively if the pleural effusion resolves after initiating a diet high in protein and carbohydrates with medium-chain triglycerides and reduced fat. A low-fat diet reduces the lymph flow to consequently reduce the chylothorax. High protein intake is recommended for surgical recovery and wound healing.

A previously healthy 8 month old presents with a history of increased sleeping, poor feeding, and one stool in the last six days. Physical exam findings include a weak cry and a lack of a social smile. The MOST likely diagnosis is: A. Hirschsprung disease B. Hypothyroidism C. Infant botulism D. Spinal muscular atrophy (SMA)

C. Infant botulism Botulism is rarely, if ever, seen in children over the age of 1 year. Most cases of infant botulism have an insidious onset. The most common symptom is constipation, while other presenting symptoms are often vague with no evidence of an infection. The classic presentation is an infant with normal sensorium but with cranial nerve palsies associated with symmetric descending weakness. Most affected infants with botulism exhibit weakness/floppiness, poor feeding, constipation, lethargy or decreased activity, poor cry, irritability, decreased respirations, and occasionally seizures. Hypothyroidism usually presents as subnormal growth. It is not associated with loss of milestones or cranial nerve palsies. Hirschsprung disease is associated with constipation but not with poor feeding or weak cry. Spinal muscular atrophy is characterized by progressive denervation of muscle with loss or regression of previous skills.

A previously healthy child presents with polyuria and recent weight loss. Pertinent laboratory results include pH 7.12, HCO3 9 mEq/L, Glucose 567 & positive ketones. Initial management strategies include: A. Infusing NS at maintenance & initiate insulin infusion B. Administering 1 mEq/kg of sodium barcarbonate & insulin bolus of 1 unit/kg C. Infusing NS fluid bolus 20 mg/kg & initiating an insulin infusion D. Administering 1 mEg/kg of potassium chloride & initiating long acting insulin therap

C. Infusing NS fluid bolus 20 mg/kg & initiating an insulin infusion Diagnosis is new onset diabetes. Diagnosis of DKA is confirmed by hyperglycemia, ketonemia, ketonuria, and metabolic acidosis (pH < 7.30, bicarbonate <15mEq/L). Initial management of DKA include fluid resuscitation with normal saline boluses, insulin infusion starting at 0.1units/kg/hr, hourly blood glucose checks, and frequent venous blood gas and electrolyte monitoring. Bicarbonate administration is contraindicated due to the risk of developing cerebral edema. Acidosis will correct over time with fluid resuscitation and insulin therapy. Bolus insulin causes the glucose level to drop too rapidly, which also increases the risk of cerebral edema.

Children are more likely than adults to experience medication errors or adverse drug events because A. Parents are more likely to report adverse effects B. Fewer formulations of medications are available C. More complex dose calculations are required D. Greater numbers of medications are used

C. More complex dose calculations are required The most significant age-specific risk for adverse events in children are related to medications. Factors that place children at increased risk for adverse events related to medications include: more complex dose calculations due to growth and physiologic maturation differences, lack of available pediatric pharmacokinetic and pharmacodynamic data compounding dosing complexity, narrow therapeutic ranges and frequent need for dose adjustment modifications, and developmental inability of the child to question appropriateness of medication or to report adverse effects.

A female adolescent presents with shortness of breath and chest pain. Past medical history is significant for gastroesophageal reflux disease and acne. Current medications include a proton pump inhibitor and oral contraceptive pills. She is afebrile and has symptoms including hypoxia, cough, & tachypnea. Which diagnostic study is MOST valuable? A. Chest ultrasound B. Electrocardiogram C. Multidector CT-PA D. Ventilation-perfusion (VQ) scan

C. Multidector CT-PA Concern for venous thromboembolism (VTE) and pulmonary embolism (PE). Oral contraceptive use is a known risk factor for VTE and PE. Presentation with PE varies widely depending on severity. Small pulmonary emboli may cause minimal or no symptoms whereas massive PE can lead to cardiopulmonary failure due to pulmonary arterial thrombi increasing afterload to the right side of the heart. Other presenting signs of PE include hypoxia, shortness of breath, cough, tachypnea, and pleuritic chest pain, which is the most common presenting sign in adolescents. While right heart strain may be evident on an electrocardiogram, and a ventilation-perfusion (VQ) scan could potentially reveal a PE, the MOST sensitive diagnostic study for suspected PE is a multidector CT-PA.

Following surgery for a complex femur fracture, an otherwise healthy adolescent develops acute respiratory distress. PRIORITY diagnostic tests include: A. Plain CXR & coagulation studies including a D-dimer B. Plain CXR & venous duplex ultrasound of affected lower extremity C. Multidetector spiral CT angiography & coagulation studies including a D-dimer D. Multidetector spiral CT angiography & venous duplex ultrasound of affected lower extremity

C. Multidetector spiral CT angiography & coagulation studies including a D-dimer The development of acute respiratory distress in an adolescent with trauma places them at high risk for a thrombosis. Acute onset of respiratory distress indicates a likelihood of pulmonary embolism (PE) as opposed to a fat embolism from the femur. The use of multidetector spiral CT angiography is the primary diagnostic approach for suspected PE, although the gold standard is pulmonary angiography, which is generally reserved for unusual cases as it is an invasive test. Obtaining a coagulation panel specifically including a D-dimer can be helpful as a normal value rules out thrombosis. However, since conditions other than thrombosis can cause an elevated D-dimer, higher levels are not necessarily diagnostically significant for thrombosis. When taken in conjunction with a multidetector spiral CT angiography that has high suspicion for a PE, an elevated D-dimer supports the diagnosis. Venous duplex ultraso

An infant presents with poor feeding and progressive lethargy. A head CT scan is obtained revealing a hyperdense crescentic mass located along the cerebral convexities. The MOST appropriate next step is to order a(n): A. Normal saline bolus B. Systemic steroid C. Neurosurgical consult D. MRI

C. Neurosurgical consult Dx is acute subdural hematoma. Initial management should focus on medically managing intracranial hypertension due to the risk for increased intracranial pressure. Neurosurgery should be consulted order to initiate urgent evacuation of the hematoma. Children with head trauma may require fluid boluses but that will be dependent on vital sign changes, which is not as imperative as urgent neurosurgical consultation and management of the increased intracranial pressure. Systemic steroids are indicated for spinal cord injury but not for closed head injury. The infant will likely require an MRI in the future for more detailed imaging and prognostication, although it is not urgent or required at this time.

Following an episode of gastroenteritis, a child is diagnosed with reactive inflammatory arthritis that is affecting joints in the lower extremities. To maximize return to the previous level of mobility, the treatment plan should include: A. Weight-bearing exercise plan B. Administration of corticosteroids C. PT & OT D. Calcium & vitamin D supplementation

C. PT & OT (Maintain joint range of motion and strength of associated muscle groups, to decrease pain, and to prevent contractures and deformities, all of which maximize the potential to return to the previous level of activity).

An adolescent presents with bilateral knee pain that has occurred for several months, exacerbated with activities such as climbing stairs and running. Pain is also worsened by sitting with the knees flexed for an extended time and is described as a grinding sensation under the kneecaps. No joint swelling is noted on examination. The MOST likely diagnosis is: A. Osgood-Schlatter disease B. Osteochondritis dissecans C. Patellofemoral pain syndrome D. Popliteal cyst (Baker's cyst)

C. Patellofemoral pain syndrome (Type of idiopathic anterior knee pain, common in adolescents. This type of knee pain worsens with activity, especially going up and down stairs, and when sitting in one position for a prolonged period of time. Treatment involves an exercise program focusing on hip girdle and vastus medialis strengthening with lower extremity flexibility)

Appropriate preventive care following splenectomy includes A. Fluconazole (Diflucan) prophylaxis daily. B. Hepatitis A, hepatitis B, and varicella vaccines. C. Pneumococcal, Haemophilus influenzae, and meningococcal vaccines. D. Sulfamethoxazole-trimethoprim (Bactrim) prophylaxis daily.

C. Pneumococcal, Haemophilus influenzae, and meningococcal vaccines. Children who require splenectomy are at risk for overwhelming postsplenectomy infection, which is associated with a 50% mortality rate. The risk increases with decreasing age of the child and is highest in the first two years following splenectomy. Asplenia also increases the risk of infection from the encapsulated organisms Streptococcus pneumoniae, Haemophilus influenzae, and meningococcus, and from Staphylococcus aureus and Escherichia coli. Vaccination with pneumococcal, Haemophilus influenzae, and meningococcal vaccines should be administered or planned around 14 days following splenectomy. Additionally, children with functional asplenia should receive daily penicillin prophylaxis.

An adolescent presents with chest pain and mild dyspnea with no additional clinical findings. The chest radiograph reveals air (increased lucency) in the mediastinum (Google image!). Which is the MOST likely differential diagnosis? A. Acute lung injury B. Foreign body aspiration C. Pneumomediastinum D. Pneumothorax

C. Pneumomediastinum In children under the age of 7 it is most often associated with a lower respiratory infection, while acute asthma is the most frequent cause in older children and adolescents. Common presenting signs and symptoms of pneumomediastinum are chest pain, dyspnea, and neck pain. Physical exam findings can include Hamman's sign, described as crunching heart sounds, or subcutaneous emphysema. As long as there is no circulatory compromise, it is usually self-limited & does not require treatment. If treatment is necessary, it focuses on the underlying cause. Analgesics such as NSAIDs may be used for chest pain. Radiographs for pneumothorax will demonstrate air in the pleural space, and those patients will typically present with chest pain, dyspnea, or tachycardia, and decreased or absent breath sounds on the side of pneumothorax. If a tension pneumothorax is present, there may also be tracheal deviation to the contralateral side.

A toddler presents with a cough and fever greater than 102° F (38.8° C) for 3 days. On physical exam, decreased breath sounds and intermittent rales are auscultated over the left lower lobe. The MOST likely diagnosis is: A. Foreign body aspiration B. Laryngotracheobronchitis C. Pneumonia D. Tracheitis

C. Pneumonia Fever & cough are the most common presenting symptoms of pneumonia. Physical exam findings include tachypnea, hypoxia, rales (crackles), rhonchi, retractions, and nasal flaring. Laryngotracheobronchitis (or croup) typically presents with hoarseness, a barking cough, and stridor and is almost always viral in origin. Lower airway adventitious breath sounds are not typically noted as this disease occurs in the upper airway. Foreign body aspiration may or may not present with a known history of the toddler having swallowed something. Clinical symptoms are often non-specific with foreign body aspiration. In smaller children, left-sided aspirations are more common than right-sided due to the bronchial anatomy, which may cause decreased or absent breath sounds over the left lung. Tracheitis is typically bacterial in origin, so a high fever may be noted; however, it otherwise presents with upper airway symptoms such as copious secretions, stridor, and respiratory distress. As wit

A neonate requiring long-term mechanical ventilation for chronic lung disease is noted to have an increased oxygen requirement without respiratory distress. A chest radiograph shows appropriate endotracheal tube position, bilateral patchy infiltrates, and subcutaneous emphysema of the neck. The neonate is MOST at risk for which of the following complications? A. Pleural effusion B. Pneumonia C. Pneumothorax D. Pulmonary interstitial emphysema

C. Pneumothorax Some of the most serious complications of assisted ventilation in the neonate are air leak syndromes, including pulmonary interstitial emphysema, pneumomediastinum, and pneumothorax. In this scenario, the neonate has a pneumomediastinum as evidenced by subcutaneous emphysema of the neck. Thus, close observation for progression to pneumothorax is crucial, as it could be life-threatening. Symptoms of pneumothorax include respiratory distress, cyanosis and hemodynamic compromise, along with tachycardia and narrow pulse pressure. While there are bilateral patchy infiltrates and an increased oxygen requirement, these alone are not diagnostic of pneumonia and could be related to other etiologies including atelectasis, aspiration, cardiogenic pulmonary edema, or pulmonary hemorrhage. A pleural effusion is a fluid collection in the pleural space that may be associated with pneumonia but on chest radiograph would appear as elevation of the diaphragm or wedge-shaped opacity.

A child presents with fever and chest pain 2 weeks after an atrial septal defect repair. Physical exam is notable for tachycardia and a pericardial friction rub. Which is the MOST likely diagnosis? A. Endocarditis B. Left ventricular dysfunction C. Post-pericardiotomy syndrome D. Residual atrial septal defect

C. Post-pericardiotomy syndrome Post-pericardiotomy syndrome most often presents 1 to 2 weeks after surgical intervention involving the myocardium. Due to a delayed presentation, patients are frequently discharged from the hospital when the symptoms occur. The common symptoms are fever and pleuritic chest pain. Heart sounds become more distant as the fluid increases. Residual ASD or VSDs may remain following primary surgical repair but are usually trivial in size and cause no hemodynamic or symptomatic changes. Left ventricular dysfunction causes CHF. Symptoms include tachycardia, respiratory distress, feeding or exercise intolerance, signs of low cardiac output, as well as growth failure. Endocarditis symptoms include a high fever and signs of hemodynamic instability.

A school-age child presents with a 3-day history of swollen eyelids and general malaise, preceded by a recent pharyngitis. Physical exam reveals a BP of 140/90 mmHg, periorbital edema, and the urinalysis is significant for moderate blood and protein. Which of the following laboratory tests will be MOST useful in making a diagnosis? A. BMP with BUN & creatinine B. CBC will differential C. Serum antistreptolysin O titer D. Urine culture

C. Serum antistreptolysin O titer The HPI is consistent with acute poststreptococcal glomerular nephritis (APSGN). Presentation of an acute nephritic syndrome typically occurs 1-2 weeks after throat infection, or 3-6 weeks after streptococcal skin infection. UOP is usually reported as decreased or dark in color. Presentation usually include hypertension, edema, & other signs of fluid overload. Diagnostic evaluation for all cases of nephritis includes an electrolyte panel, creatinine, BUN, CBC with differential, urinalysis and culture, and throat culture. The diagnosis of APSGN cannot be confirmed unless there is clear evidence of a prior streptococcal infection. A rising antibody titer to streptococcal antigen(s) (ASO) confirms a recent streptococcal infection. While CBC and BMP are included in the workup, they are not specific enough for making this diagnosis. A urine culture is also part of the workup but would only diagnose other urinary tract infection organisms and is not speci

An adolescent with a BMI of 30 presents with vague, colicky, right upper quadrant pain, nausea, and vomiting. Abdominal ultrasound reveals cholecystic thickening. Which of the following behavioral modifications should be implemented? A. Start a strict elimination diet to avoid potential food intolerances B. Initiate a diet & exercise regimen to promote rapid weight loss C. Utilize weight loss counseling to reduce weight gradually over time D. Maintain a food journal to track & encourage fiber &

C. Utilize weight loss counseling to reduce weight gradually over time Management for acute cholecystitis can either be supportive or surgical via laparoscopic cholecystectomy. Supportive management includes hospital admission with gastric decompression, intravenous fluids, and broad-spectrum antibiotics. The behavioral modification that should be encouraged to prevent future episodes of cholecystitis is weight loss. This should be done in a controlled manner with involvement of nutritionists and weight loss counselors. Rapid weight loss should be discouraged as it can induce stone formation and will likely not provide for long term results. An increase in consumption of fiber and fluids would be helpful in preventing constipation, not cholecystitis. Tracking food intolerances may be helpful for prevention of abdominal pain associated with gastroesophageal reflux, food allergies, and other causes of food intolerance, but will not prevent cholecystitis.

A 16 year old presents with a 6-hour history of migraine headache unresponsive to ibuprofen. With a normal initial physical exam, which of the following is the MOST appropriate next step? A. obtain EEG B. obtain neuroimaging C. administer sumatriptan D. administer topiramate

C. administer sumatriptan Goals of migraine treatment are to abort the migraine pain with as minimal side effects as possible. Multiple medications are used in treatment of pediatric and adolescent migraine; few have significant data and fewer have FDA approval. First-line treatment is typically NSAIDs and acetaminophen; if these are ineffective, triptans are commonly employed. Triptans with FDA-approval in pediatric patients include: almotriptan (ages 12 years and older), rizatriptan (Maxalt) (in children and adolescents 6-17 years of age), sumatriptan (ages 12 years and older), and zolmitriptan (ages 12 years and older). With sumatriptan demonstrating the best pain relief in the studies. Rizatriptan is the only medication approved for children under 12 years of age. Topiramate (Topamax) is used off-label as a prophylactic agent and would not be effective for acute treatment. Neither neuroimaging nor EEG is indicated in this child with a normal physical exam.

A previously healthy infant presents with three days of rhinorrhea, low-grade fever, and cough. Vital signs include temp 38.4 (101.1), HR 170, RR 58, BP 70/48, O2 89% on RA. On physical examination, mild to moderate intercostal and subcostal retractions are noted with diffuse crackles and wheezes auscultated over all lung fields. The MOST likely diagnosis is: A. asthma B. bacterial pneumonia C. bronchiolitis D. viral pneumonia

C. bronchiolitis On auscultation, bronchiolitis presents with diffuse (non-focal) wheezes and crackles. Depending on the severity, hypoxia may or may not be present and respiratory distress can be absent to severe. Respiratory syncytial virus is the most commonly associated virus with bronchiolitis. Asthma presents with bronchial obstruction and hyperresponsiveness. This results in diffuse wheezing or, if severely obstructed, decreased aeration. Crackles indicate the accumulation of mucus or fluid, which typically occurs with bronchiolitis or pneumonia rather than asthma . Bacterial pneumonia is a focal lung infiltrate resulting in localized crackles on auscultation. Wheezing is uncommon with an isolated bacterial pneumonia. Viral pneumonia is principally caused by Mycoplasma pneumoniae and is more commonly seen in school-age children rather than the infant age group. Signs and symptoms, such as fever, cough, and fatigue, may be similar to those of bacterial pneumonia but in a milde

A school-age child sustains a grade II renal injury. During the acute period of non-operative management, which of the following serial laboratory values is MOST important to monitor? A. blood urea nitrogen (BUN) and creatinine B. creatine kinase C. hemoglobin and hematocrit D. white blood cell count (WBC) with differential

C. hemoglobin and hematocrit The most common sign of trauma to the kidney is hematuria, but the degree of hematuria does not indicate the degree of injury. The majority of pediatric renal injuries can be managed non-operatively. All grades require admission for IV hydration, serial hemoglobin and hematocrit levels, monitoring for hypertension, and resolution of hematuria. Grade III and IV injuries involve laceration and require repeat imaging within 24 to 48 hours to evaluate kidney perfusion, presence of extravasation, and size of hematomas. Limited activity is required for 2-4 weeks after gross hematuria is resolved, and return to regular activity is permitted only after microscopic hematuria has resolved.

A child newly diagnosed with an intracranial mass is intubated and sedated and acutely develops hypertension, bradycardia, and anisocoria with a fixed, dilated left pupil. The MOST appropriate intervention is administration of: A. epinephrine B. hydralazine C. hypertonic sodium chloride D. sodium bicarbonate

C. hypertonic sodium chloride Hypertension, bradycardia, pupillary dilatation, and posturing are signs and symptoms of herniation, which is a medical emergency. Immediate pharmacologic intervention includes hyperosmolar therapy with hypertonic sodium chloride or mannitol. Hypertonic saline is dosed as either a bolus of 6 to 10 mL/kg or as a continuous infusion in a range of 0.1 to 1 mL/kg/hour. Theoretical beneficial effects include restoration of normal cellular resting membrane potential and cell volume, inhibition of inflammation, stimulation of arterial natriuretic peptide release, and enhancement of cardiac output. As the hypertension and bradycardia are symptoms of herniation, management with epinephrine or hydralazine is not appropriate. Sodium bicarbonate is indicated for documented acidosis when adequate ventilation is assured.

A child presents with an isolated complaint of bleeding gums and multiple bruises approximately 3 weeks after a viral respiratory infection. Complete blood count (CBC) results include WBC 8,400, Hgb 11, Hct 38% & Platelets 95,000. The MOST likely diagnosis is: A. acute lymphocytic leukemia B. hemolytic uremic syndrome C. immune thrombocytopenic purpura D. Wiskott-Aldrich syndrome

C. immune thrombocytopenic purpura Thrombocytopenia is defined as a platelet count below 150,000/mm3. Immune thrombocytopenic purpura is an autoimmune process that typically presents 1-4 weeks following a viral infection in otherwise healthy children. The classic presentation includes sudden appearance of multiple bruises and bleeding of the gums. Physical exam findings are normal other than bruises, petechiae and perhaps purpura. Leukemia characteristically presents with reductions in all cell lines in the CBC and a history of other significant findings such as fatigue, anorexia, and intermittent fever. Hemolytic uremic syndrome (HUS) is a thrombotic microangiopathy that presents with the classic triad of hemolytic anemia, thrombocytopenia, and renal compromise. HUS occurs most commonly after an acute diarrheal illness due to Escherichia coli, but may also follow other common childhood infections including influenza, human herpes virus 6, and parvovirus B19. Wiskott-Aldrich syndrom

A previously healthy school-age child presents with confusion, enuresis, and polydipsia. Initial laboratory studies demonstrate severe dehydration, metabolic acidosis, and ketonuria. The PRIORITY in the first hour of treatment is to A. replace electrolytes. B. reverse the acidosis. C. initiate fluid resuscitation. D. correct the hyperglycemia.

C. initiate fluid resuscitation. Diagnostic evaluation for DKA begins with laboratory evaluation to include: electrolytes, BUN, creatinine, pH analysis (venous or arterial sample), serum ketones, urinalysis, osmolality, and hemoglobin A1C. Primary diagnostics consistent with DKA include serum glucose greater than 200 with ketoacidosis, ketones in serum and urine, pH less than 7.3, and serum bicarbonate less than 15. The priority in the first hour is to initiate fluid resuscitation. During the first hour, an initial fluid bolus of 10-20 ml/kg of normal saline should be administered with frequent reevaluation of fluid and neurologic status. In the second hour, the aim is correction of acidosis, reversal of ketosis, restoration of electrolyte balance, and slow, gradual correction of hyperglycemia with ongoing reversal of dehydration. Insulin is typically started in the second hour of therapy. The risk for cerebral edema is highest 4-12 hours into treatment; it is the most serious compli

Regular ophthalmology screening for uveitis is required for a child with which of the following diagnoses? A. dermatomyositis B. Henoch-Schönlein purpura C. juvenile idiopathic arthritis D. Legg-Calvé-Perthes disease

C. juvenile idiopathic arthritis Iridocyclitis, a form of chronic anterior uveitis, is one of the more common and serious complications associated with juvenile idiopathic arthritis (JIA). Routine ophthalmologic screening with slit lamp examination is imperative for early diagnosis. Children with oligo- or polyarticular arthritis with onset of disease before age 7 years and positive antinuclear antibodies are at highest risk and require screening every 3 months. Children with systemic JIA have the lowest risk of uveitis and require screening annually. Ophthalmologic abnormalities associated with juvenile dermatomyositis include cataracts, which are most frequently corticosteroid induced. Retinitis has been reported with juvenile dermatomyositis but is unusual. Henoch-Schönlein purpura and Legg-Calvé-Perthes disease are not typically associated with ophthalmologic complications.

A 2 year old with previously undiagnosed, uncomplicated coarctation of the aorta has normal growth and development. The stenosis is located just distal to the left subclavian artery. Medical management includes A. diuretics to control pulmonary venous congestion. B. serial cardiac catheterizations for anatomic assessment. C. monitoring upper extremity blood pressures for hypertension. D. activity restriction based on echocardiograph assessment of left ventricular hypertrophy.

C. monitoring upper extremity blood pressures for hypertension. Infants with significant obstruction may feed poorly, or have respiratory symptoms or signs of circulatory shock. Those with less obstruction or adequate collateral circulation may be asymptomatic. In fact, more are asymptomatic than symptomatic. Asymptomatic infants and children grow and develop normally. Their lower extremity pulses are weak or absent, and they have upper extremity hypertension. Normally, the systolic blood pressure in the lower extremity is 5 to 10 mmHg higher than that in the arm; those with COA have leg systolic pressures equal to or lower than those in the arm. Medical management of asymptomatic children with COA includes close monitoring for hypertension in the arm for increasing pressure differences between the arm and leg. Cardiac catheterizations are not routinely done for anatomic assessment as they are invasive but can be used as a primary intervention. MRI or CT with contrast and 3-dimensiona

An adolescent with no significant past medical history who could not be wakened by caregivers is being evaluated. There is no history or physical findings of trauma, and pupils are pinpoint bilaterally. Vital signs and Glasgow Coma Scale score are: HR 60, RR 8, BP 80/54 (62), T 36.2, O2 86% on RA, GSC - Eye opening 2, verbal 3, motor 4 (total 9). The MOST likely diagnosis is: A. anticholinergic ingestion B. meningitis C. opiate intoxication D. space occupying lesion

C. opiate intoxication A depressed level of consciousness is the consequence of insult to the brain. In this scenario, trauma is ruled out based on history and physical assessment. The classic presentation of opioid toxicity includes respiratory and central nervous system depression accompanied by miosis. Thus, in the case presented, bradypnea, oxygen desaturation, pinpoint pupils, and depressed level of consciousness in an adolescent with no previous health concerns or evidence of trauma is highly concerning for opiate intoxication. Anticholinergic toxicity can also present with coma; however, other cardinal findings in the anticholinergic toxidrome include mydriasis, fever, tachycardia, and hypertension. These signs are described in the classic mnemonic "hot as a hare, blind as a bat, dry as a bone, red as a beet, and mad as a hatter". A space occupying lesion, such as an intracranial bleed or tumor, causes shifting of brain tissue and an increase in intracranial pressure, leading

An adolescent is evaluated for a 2-month history of pain and swelling of the left thigh. A radiograph reveals a lesion within the femur which has a sunburst pattern. The MOST likely diagnosis is: A. osteochondroma B. osteomyelitis C. osteosarcoma D. rhabdomyosarcoma

C. osteosarcoma Typical presenting complaints are pain or swelling, which is often initially attributed to sports or overuse injuries. The most frequent location is in the metaphysis of long tubular bones, at the site of the most rapidly growing growth plates. Routine blood counts and chemistries are normal, but lactate dehydrogenase or alkaline phosphatase levels may be increased. Radiographs classically reveal a "sunburst" or "starburst" pattern. Osteochondromas are most often benign tumors that arise from the metaphysis of long bones. These are asymptomatic and discovered when irritated by sports or other activities. Radiographically, osteochondroma appears as a "stalk" off the surface of the bone, typically in a direction away from the adjacent joint. Rhabdomyosarcoma commonly develops in the extremities or bone marrow and occurs most often in adolescents or young adults. Radiographs are very non-specific; other imaging modalities including magnetic resonance imaging or nuclear m

In which clinical scenario is a platelet transfusion MOST appropriate? A. platelet count of 24,000/mm3 in a child with immune thrombocytopenic purpura and bleeding from the gums whose initial platelet counts were below 10,000/mm3 B. platelet count of 38,000/mm3 in a child with hemolytic uremic syndrome and no clinical bleeding C. platelet count of 72,000/mm3 in a child who is immediately status-post surgical evacuation of an epidural hematoma D. platelet count of 83,000/mm3 in a child with hepar

C. platelet count of 72,000/mm3 in a child who is immediately status-post surgical evacuation of an epidural hematoma The goal of platelet transfusion is to stop active hemorrhage or to prevent bleeding. Platelet counts below 10,000/mm3 are associated with spontaneous intracranial bleeding and should, therefore, be treated with platelet repletion. A platelet transfusion is contraindicated with heparin-induced thrombocytopenia due to an increased thrombotic risk. Platelet transfusion is avoided with a diagnosis of immune thrombocytopenic purpura once initial counts are greater than 20,000/mm3, as long as there is no intracranial hemorrhage or other identified source of ongoing blood loss. Platelet transfusion is also avoided with hemolytic-uremic syndrome unless there is a risk for severe hemorrhage. For children with a decreased platelet production, such as that seen with bone marrow suppression following chemotherapy, a prophylactic platelet transfusion is indicated when the pla

An adolescent is evaluated for recent syncopal episodes that have occurred when the school bell rings to signal a change of class. Which of the following 12-lead electrocardiogram (ECG) findings are expected for the suspected underlying diagnosis? A. delta wave B. prolonged PR interval C. prolonged QT interval D. widened QRS interval

C. prolonged QT interval An ECG should be obtained as part of the evaluation of all syncopal episodes to evaluate for a dysrhythmia such as long QT syndrome, Wolff-Parkinson-White syndrome (WPW), or ventricular hypertrophy. A long QT interval occurs in long QT syndrome (LQTS), a disorder characterized by delayed ventricular repolarization. Diagnosis is based on ECG findings in conjunction with specific signs and symptoms, along with a careful family history.Signs and symptoms associated with long QT interval on ECG include seizures, syncope, palpitations, and dizziness or in some cases, cardiac arrest. Symptoms most often coincide with exercise, strong emotion, or sudden auditory stimuli. In those with marked prolongation of the QT, ventricular tachycardia (VT) may occur in the form or torsades de pointes, which means "twisting of the points". This distinct form of VT, described as polymorphic, is characterized by progressive changes in the amplitude and polarity of the QRS complex.

Law enforcement authorities request information regarding visible injuries assessed in a 13-year-old victim of sexual abuse. What is the MOST appropriate action to take? A. call the parent to obtain consent B. refer the request to public relations C. provide the information as requested D. require a court order for release of information

C. provide the information as requested In most jurisdictions, sexual abuse is a criminal offense and is reported to law enforcement. When the victim is a child, a civil report to the local child protection agency is also required. Protocols for reporting to child protection and law enforcement agencies should be developed according to state laws and included in the facility's sexual assault procedure. The parent or caregiver should be informed if a civil (child abuse) report and/or criminal (police) report is being made. The provider or social worker must explain the practical consequences of the reports to the parent or caregiver. Once the suspicion of abuse has been established and consultations obtained, the next step is reporting. Laws dictate how reports are to be made and to whom. Although laws vary from state to state, most have common elements. Providers should be familiar with his or her current state law and with institutional policy. Laws governing reporting define abuse a

An adolescent female with a body-mass index of 30 has complaints of severe headaches that have occurred when going to the bathroom followed by brief loss of vision and hearing own heartbeat. The MOST likely diagnosis is: A. cluster headache B. neuroblastoma C. pseudotumor cerebri D. tension headache

C. pseudotumor cerebri Pseudotumor cerebri is a disorder characterized by increased intracranial pressure. It is associated with certain medical conditions and typically affects adolescent females, with a higher frequency in adolescents who are overweight. The symptoms of pseudotumor cerebri include nonspecific headaches, pulsatile tinnitus, malaise, photophobia and phonophobia. Immediate referral and work-up with a lumbar puncture are important for management. Cluster headaches are vascular headaches and occur in clusters over a period of weeks or months. Pain is severe and comes on suddenly. Tension headaches typically cause mild to moderate pain and are easily relieved by acetaminophen or ibuprofen. Neuroblastoma is an extracranial tumor which most often occurs in children younger than the age of 2.

A previously healthy adolescent presents with a sudden onset of severe right hip and knee pain. Physical examination findings include limited internal rotation of the right hip and inability to bear weight. What is the MOST appropriate diagnostic study? A. nuclear medicine bone scan of pelvis, right femur, and knee B. magnetic resonance imaging (MRI) of right hip, femur, and knee C. radiographs of pelvis in anterior-posterior and lateral views D. ultrasound of right hip

C. radiographs of pelvis in anterior-posterior and lateral views The differential diagnosis of hip pain in children and adolescents includes fractures, Legg-Calvé-Perthes disease, slipped capital femoral epiphysis (SCFE), septic arthritis, and toxic synovitis. In the adolescent age group, sudden onset of pain without history of trauma or fever is most representative of slipped capital femoral epiphysis.

A 5 month old is evaluated for emesis, blood-streaked stools, and abdominal tenderness. Which of the following information from the past medical and surgical history is MOST critical? A. born at 30 weeks gestation B. hydrolyzed formula initiated for milk protein allergy C. required placement of peritoneal drain for necrotizing enterocolitis D. previously obtained rectal biopsy confirmed presence of ganglion cells

C. required placement of peritoneal drain for necrotizing enterocolitis Necrotizing enterocolitis (NEC) is characterized by ischemic necrosis of the intestinal mucosa. It is commonly treated medically with bowel rest and antibiotic therapy. More severe cases require surgical management with laparotomy and bowel resection or peritoneal drain placement. Infants with a history of NEC are at risk of developing intestinal strictures, occurring in approximately 10% of children independent of NEC severity or whether it was treated medically or surgically. Presenting symptoms can include bloody stools, failure to thrive, and signs of bowel obstruction including vomiting. Prematurity is not a risk factor for vomiting. Hydrolyzed formula is generally well tolerated by infants diagnosed with allergy to cow's milk protein and while milk protein allergy can cause vomiting, treatment with hydrolyzed formula should remedy the problem. A normal rectal biopsy will have ganglion cells present. The ab

A 4 year old is being scheduled for an adenotonsillectomy. A history of which of the following MOST warrants an echocardiogram prior to the procedure? A. hypoxic event B. pulmonary edema C. right ventricular hypertrophy D. ST segment depression

C. right ventricular hypertrophy Adenotonsillectomy is the recommended first-line treatment for most children with obstructive sleep apnea syndrome (OSAS). Children with severe OSAS can exhibit symptoms of failure to thrive, cor pulmonale, and developmental delay or behavioral problems. In addition, asymptomatic pulmonary hypertension and systemic hypertension can occur as a result of this problem. Echocardiography should be performed prior to anesthesia and surgery to evaluate for right ventricular hypertrophy as a complication of chronic OSAS.

A 2 month old with an infantile hemangioma around the left eye is being evaluated for propranolol therapy. Which of the following findings would be considered a potential CONTRAINDICATION? A. prematurity B. hypertension C. sinus bradycardia D. family history of diabetes

C. sinus bradycardia Propranolol is a beta-adrenergic blocker currently being used for the treatment of infantile hemangioma (IH). The exact mechanism of action for propranolol on IH is not well understood, but hypothesized mechanisms include vasoconstriction, decreased renin production, inhibition of angiogenesis, and stimulation of apoptosis. Propranolol has both the negative chronotropic and inotropic cardiac effects resulting in decreased heart rate and blood pressure. Due to the negative chronotropic effects, children with documented bradycardia are not candidates for treatment with propranolol.

A child currently receiving induction chemotherapy for acute lymphocytic leukemia is noted to have a positive Chvostek and Trousseau's sign on physical exam. Which complication is MOST likely represented by these findings? A. hyperleukocytosis B. superior vena cava syndrome C. tumor lysis syndrome D. typhlitis

C. tumor lysis syndrome Tumor lysis syndrome (TLS) occurs when rapid lysis of malignant cells abruptly releases large amounts of cellular components into the bloodstream. Resulting metabolic derangements include hyperuricemia, hyperkalemia, hyperphosphatemia, and hypocalcemia. The most serious complication of hyperkalemia is potentially fatal arrhythmia. Hyperphosphatemia causes increased calcium precipitation in the kidneys; this precipitation combined with kidney insufficiency results in hypocalcemia. Chvostek's sign and Trousseau's sign are physical exam findings of hypocalcemia. Chvostek's sign is positive when a tap on the facial nerve located anterior to the ear lobe and below the zygomatic arch produces twitching or contraction of the facial muscles. Trousseau's sign is positive when carpopedal spasm, wrist/metacarpophalangeal thumb flexion, and hyperextension of the fingers occur after inflation of a blood pressure cuff to suprasystolic pressure for several minutes.

The palliative care team is being consulted for a 6 month old with complex medical conditions who has been hospitalized since birth. It is MOST important for the family to understand that the primary focus of palliative care is to: A. Assess the infant, modify care goals & focus on discharge preparation B. Assist with anticipating bereavement needs & transition to hospice care C. Prevent readmissions to the hospital for acute illnesses after the infant is discharged to home D. Develop desirable

D. Develop desirable goals of care, recommend life prolonging therapies & enhance quality of life

A toddler presents with vomiting and altered mental status after an ingestion of a large amount of aspirin about 45 minutes ago. Prior to the administration of activated charcoal, the PRIORITY is to: A. Obtain a urine drug screen B. Ensure IV access C. Obtain and EKG D. Ensure adequate airway protection

D. Ensure adequate airway protection

Which of the following BEST exemplifies the principle of beneficence? A. Double-checking the dosage of an antibiotic before prescribing B. Parental consent is given for child's treatment upon admission C. Adjusting the dosage of antibiotic based on the child's renal function D. Prescribing antibiotics for a child with bacterial pneumonia

D. Prescribing antibiotics for a child with bacterial pneumonia (Beneficence is to provide care that benefits the child Double-checking the dosage of antibiotics & adjusting medication dosing are examples of nonmaleficence, which is the concept of avoiding harm)

Which of the following exam findings in a child are MOST consistent with a diagnosis of Stage III hepatic encephalopathy? A. Periodic lethargy, mood swings, incontinence & hyperreflexia B. Drowsiness, confusion, agitation, & incontinence C. Asterixis, agitation, response limited to noxious stimuli, areflexia D. Confusion, arousable from stupor, asterixis, hyperreflexia

D. Confusion, arousable from stupor, asterixis, hyperreflexia STAGE I: alertness, agitation, but distractible; infants and young children are irritable and fussy. Reflexes are normal but tremor with poor handwriting is noted. Children are able to obey age-appropriate commands. STAGE II: confusion and lethargy, combativeness or inappropriate euphoria. Reflexes are hyperactive with asterixis present, and while movements are purposeful, the child may not be able to obey commands. STAGE III: stuporous but arousable, motor response to pain, incoherent speech, hyperreflexia, hyperventilation, and asterixis is present. STAGE IV: unconsciousness and either unresponsive or responds non-purposefully to noxious stimuli, hyperreflexia, irregular respirations, and sluggish pupillary response.

A previously healthy preschooler who is fully immunized, presents with a history of fever, URI symptoms, & joint pain over the past several days. Physical exam reveals petechiae and hepatosplenomegaly. Vital signs include HR 156, RR 32, temp 101.6 (38.7), BP 86/44 & O2 98% on RA. Lab results include Na+ 132, K+ 6, Glucose 100, Ca+ 0.95, Phos 6.3, WBC 105,000, Hgb 6.1 & Plt 10,000. Which is the BEST course of action? A. Obtain LFTS & type & cross for blood products B. Administer calcium gluconat

D. Administer a fluid bolus & consult oncology (The combination of leukocytosis, thrombocytopenia, and anemia should raise red flags for an oncologic process, most likely leukemia. After initial laboratory values are obtained, the child should receive aggressive hydration.)

An adolescent with a chronic illness is preparing to transition from pediatric healthcare providers to adult care. Which of the following would MOST likely promote a successful transition? A. Expect adult care providers to educate adolescent about self-care management B. Readiness assessment should begin a year prior to the planned transfer C. Allow the adolescent & family to lead the coordination effort D. Care coordination is initiated by the team currently providing pediatric care

D. Care coordination is initiated by the team currently providing pediatric care A medical transition is defined as "the purposeful, planned movement of adolescents and young adults with chronic physical and medical conditions from child-centered to adult-oriented health care systems." "Transition" is very different from "transfer" in that a transition is intended to be an anticipated and coordinated effort, whereas a transfer is only one component of the transition. A key feature of the guidelines is the outlining of 6 core elements for transition from pediatric to adult care. These include the pediatric practice initiating the discussions and planned transition of care, assessing readiness for transition early and often, and initiating self-management discussions around age 14 years, not upon transfer. Care coordination is key, and although the child and family are integral and must participate in this aspect, they are not solely responsible for managing the care coordination.

In a child presenting with severe hyponatremia, to rapidly terminate acute seizure activity the serum sodium must be corrected to at least: A. 118 mEq/L B. 120 mEq/L C. 123 mEq/L D. 125 mEq/L

D. 125 mEq/L Low serum sodium concentrations are associated with seizures and coma. When a child with severe hyponatremia is seizing, treatment should begin immediately to acutely raise the sodium level to 125 mEq/L in an effort to terminate seizure activity. This is accomplished by administering IV hypertonic sodium chloride (3% NS) bolus over 15-20 minutes. If hypertonic saline solution is not available, normal saline (0.9%) can be given as a 20 mL/kg bolus dose. Once acute correction is achieved, the continued correction should be at a slower rate (0.5 -1 mEq/L per hour) to avoid rapid water shifts from intracellular to extracellular compartments that can produce neurologic complications, such as intracranial bleeding.

After an occurrence of nephrolithiasis, which intervention is MOST importantly included in education provided to the adolescent and family related to long-term prevention of this condition? A. Taking a citrate supplement B. Adhering to a low sodium diet C. Taking a daily diuretic medication D. Ensuring at least 2 liters of fluid intake daily

D. Ensuring at least 2 liters of fluid intake daily Nephrolithiasis (kidney stones), are due to congenital and structural urologic abnormalities, hypercalciuria, hyperoxaluria/oxalosis, hypocitraturia, or other metabolic abnormalities. Management is centered around pain control, urine culture with treatment of concomitant UTI, and hydration. Urologic intervention such as extracorporeal shock wave lithotripsy or percutaneous nephrolithotomy may be necessary in high risk cases (e.g., single kidney) or cases of unrelenting pain or urinary obstruction.

In a previously healthy, immunized infant who has a urinalysis positive for leukocyte esterase and nitrates, which of the following findings is MOST supportive of a diagnosis of pyelonephritis? A. Lethargy B. Malodorous urine C. Vomiting 24 hours prior D. Fever of 102.6 F (39.2 C)

D. Fever of 102.6 F (39.2 C) Pyelonephritis should be suspected in any child with findings of UTI accompanied by high fever. The pathogens most commonly associated with pyelonephritis include Escherichia coli, Klebsiellaspecies, Proteusspecies, Pseudomonas aeruginosa, Enterococcus species, Serratia species, and Staphylococcus aureus. Infants with pyelonephritis can present with strong or unusual smelling urine but may also have non-specific symptoms such as poor feeding, irritability, and jaundice. These patients are often ill appearing and a fever greater than 102.2˚ F (39˚ C) is common. In infants younger than 1 year who are ill-appearing, or when pyelonephritis is suspected, additional laboratory studies include CBC, CRP, ESR, BUN and creatinine. Inpatient admission is warranted for these infants for administration of IV antibiotics and hydration. Sepsis is common in infants with pyelonephritis; thus, when suspected, blood cultures are indicated and should be obtained before sta

Which of the following is the MOST likely presentation of behavioral and psychiatric disorders in a preschooler? A. Fear of the dark B. Issues with identity formation C. Learning disabilities D. Frequent temper tantrums

D. Frequent temper tantrums

In a child who is dependent on mechanical ventilation, findings suggestive of a tracheal plug include low tidal volumes and: A. Low RR & high peak inspiratory pressure (PIP) B. Low RR & low PIP C. High RR & low PIP D. High RR & high PIP

D. High RR & high PIP Tracheal plugging can occur is due to thick mucosal secretions obstructing the tracheostomy tube. In order to overcome the obstruction, the ventilator will try to deliver higher pressures (high PIP) but will meet resistance, resulting in low tidal volumes. Both the ventilator & patient will try to increase respiratory rate in an attempt to provide adequate oxygenation and ventilation. The DOPE pneumonic is helpful in systematically evaluating possible causes for a ventilation problem. "D" stands for displacement. (Although more typical for an oral endotracheal tube, a tracheostomy tube can displace outside of the trachea and into the subcutaneous space.) The "O" stands for obstruction; "P" for pneumothorax; and "E" for equipment.

A 3 month old was witnessed falling from a bed to the floor by a caregiver. Which component of the history is MOST important? A. Developmental history & achievement of milestones B. Newborn history & post-natal growth assessment C. Social history including caregiver identification D. History of the event leading to presentation

D. History of the event leading to presentation Comprehensive history taking is a critical part of a trauma assessment. For an infant or young child presenting following trauma, the most important component is the history of the event itself.

Following resection of an astrocytoma, a child develops decreased urine output. Laboratory results include blood Na+ 129 & osmolarity 250, and urine Na+ 40, Osmolarity 450 & specific gravity 1.050. Which of the following therapies would be MOST appropriate to initiate? A. fluid bolus 20 ml/kg of NS B. IV fluid D5 1/2 NS at 1.5 maintenance C. vasopressin infusion D. IV fluids D5 NS at 2/3 maintenance

D. IV fluids D5 NS at 2/3 maintenance Syndrome of inappropriate antidiuretic hormone (SIADH) is a common cause of hyponatremia in hospitalized children. It most often occurs in children with central nervous system or pulmonary diseases and can be induced by some medications. Hallmark findings include hyponatremia with decreased urine output, and serum osmolarity and increased urine osmolarity, urine sodium, and urine specific gravity. Another significant characteristic of SIADH is hypervolemia with signs of fluid overload. Differentiation of SIADH from cerebral salt wasting is clinically important as treatment differs significantly. A key differentiating factor is that urine output is decreased in SIADH, but normal to slightly increased in cerebral salt wasting. Urine sodium is significantly higher in cerebral salt wasting than in SIADH. Treatment of SIADH is initially aimed at correction of sodium. Severe hyponatremia with levels below 120 mEq/L may present with seizures and require

An otherwise healthy child presents with upper extremity weakness and the inability to stand following a one-week history of progressive ataxia. The caregiver reports that the child had an upper respiratory infection 2 weeks ago. On physical exam, ascending motor weakness and areflexia are noted. Which of the following is the MOST anticipated treatment? A. Antibiotics B. Antitoxin C. Corticosteroids D. IVIG

D. IVIG With these progressive symptoms after a recent viral infection, Guillain-Barré Syndrome (GBS) is the most likely among those in the differential list. GBS is associated with ascending weakness and areflexia. IVIG is the preferred treatment given its ease of administration and equivalency to plasmapheresis. It is given daily for 5 days at a dose of 0.4gm/kg/day.

A one month old who presents with severe hypotonia, feeding difficulties, hypogonadism, and abnormally small hands and feet is determined to be at risk for failure to thrive. The MOST likely underlying diagnosis is: A. Fetal alcohol syndrome B. Fragile X syndrome C. Noonan syndrome D. Prader-Willi syndrome

D. Prader-Willi syndrome Findings of Prader-Willi syndrome includes severe hypotonia, feeding difficulties (which causes risk of FTT), intellectual delays, hypogonadism, abnormally small hands and feet, & dysmorphic features. Later, children develop hyperphagia and are at risk for morbid obesity. Noonan syndrome is characterized by short stature, developmental delay, and cardiac defects with the latter being the most detrimental, usually requiring surgical intervention. Fragile X syndrome is the most common inherited form of intellectual disability Fetal alcohol syndrome also causes developmental delay & growth retardation. None of these other syndromes are as highly associated with FTT, especially in infancy, as Prader-Willi syndrome.

An afebrile school-age child presents with a painful, erythematous, fluctuant lesion on the arm that measures 1.5 cm in diameter. Appropriate treatment includes: A. Topical antibiotic ointment & covering lesion with a dressing B. Empiric PO antibiotics with office f/u in a week C. Washing daily with soap and water and keeping dry D. Incision & drainage with Gram stain of fluid

D. Incision & drainage with Gram stain of fluid For purulent skin soft tissue infections, I&D is indicated, specifically in mild cases lacking systemic symptoms. Sending the drainage for Gram stain and culture is recommended. The choice to prescribe antibiotics for S. aureus should be limited to those patients with moderate to severe purulent infections with symptoms of SIRS, such as fever, tachypnea, tachycardia, or WBC count elevated over 12,000, or for any patient with a nonpurulent skin infection such as cellulitis, necrotizing fasciitis, or erysipelas. Antibiotics should be considered for children with impairment of host defenses, such as those who are immunocompromised. If antibiotics are prescribed, they should be prescribed after I&D & f/u should occur in 48 hours to assess efficacy of treatment. Following I&D, simply covering the site with a dry dressing is usually the easiest and most effective treatment. Topical antimicrobials are not recommended for the treatment of mild

A toddler's complete blood count (CBC) reveals the following results: WBC 11, RBC 33, Hct 28%, Hgb 9, MCV 60, Plt 200, RDW 18%, Reticulocytes <0.1 Which type of anemia is the MOST likely? A. Aplastic anemia B. Diamond-Blackfan anemia C. Glucose-6-phosphate dehydrogenase (G6PD) deficiency D. Iron-deficiency anemia

D. Iron-deficiency anemia In a toddler, anemia is defined as a hemoglobin <12.0 g/dl. Types of anemia are classified and diagnosed by reticulocyte count and size of red blood cells (microcytic, normocytic, and macrocytic). Iron-deficiency anemia is defined as microcytic anemia with a low reticulocyte count and an elevated red cell distribution width. Diamond-Blackfan anemia is a rare congenital form of hypoplastic anemia. It causes constitutional bone marrow suppression with resultant severe normochromic macrocytic anemia, leukopenia, and thrombocytopenia. Aplastic anemia is a life-threatening disease of bone marrow failure with CBC revealing pancytopenia. Glucose-6-phosphate dehydrogenase (G6PD) deficiency is a hemolytic anemia, characterized by severe anemia (hemoglobin less than 2.5), leukocytosis, reticulocyte count greater than 30%, and large red cell distribution width (RDW).

A child with leukemia requires intrathecal chemotherapy. The desired intervertebral space for instillation of the medication(s) is: A. T1-T2 B. T8-T9 C. L1-L2 D. L4-L5

D. L4-L5 The lumbar vertebral spaces of L3-L4 or L4-L5 are the desired intervertebral space. In drawing an imaginary line between the left and right iliac crests, L4 will be crossed. The spinal cord terminates above this level, leaving the cauda equina in the spaces L3-L5.

A 16 month old with Trisomy 21 presents with a 4 day history of low grade fever, pallor and decreased activity. Laboratory values reveal hgb 5.2, hct 14%, plt 43,000, reticulocyte count 2.2%, pt 13 sec, ptt 33 sec & wbc 1,700 Which is the MOST likely diagnosis? A. Bacterial sepsis B. Fanconi anemia C. Hemolytic uremic syndrome (HUS) D. Leukemia

D. Leukemia Children with Trisomy 21 have a higher incidence of developing leukemia, most often occurring in the early years of life. This predisposition for developing leukemia is thought to be related to the abnormal genetic karyotype. Clinical findings of leukemia include persistent fevers, anemia, thrombocytopenia and either hyperleukocytosis or leukopenia.

An adolescent male with a several month history of groin pain presents after feeling something "pop" while playing basketball. Physical exam shows a BMI of 28 and obvious limping with weight-bearing. Which of the following pelvic radiologic studies is MOST appropriate? A. Bone scan B. CT scan C. MRI D. Plain radiograph

D. Plain radiograph Slipped capital femoral epiphysis (SCFE) is a condition in which there is posterior and inferior slippage of the proximal femoral epiphysis (on the femoral neck), separating the epiphyseal (growth) plate from the femoral head. There is a higher incidence in children who are overweight or obese and during growth surges. Radiographs in the AP & lateral views are necessary for diagnosis and provide good visualization of the femoral head of both hips. The lateral view allows for observing posterior displacement of the femoral epiphysis, which is diagnostic of SCFE. CT scan may be used to assess severity & MRI should only be used when radiographic findings are unclear. There is no role for nuclear medicine scans in making the diagnosis.

A child with a history of congenital heart disease fell while playing and presents with a closed fracture of the humerus. Physical exam reveals mild bradycardia and a moderate pain score. Which of the following medications would MOST likely explain this child's bradycardia? A. Enalapril (Vasotec) B. Furosemide (Lasix) C. Levothyroxine (Synthroid) D. Propranolol (Inderal)

D. Propranolol (Inderal) (Decreases heart rate, myocardial contractility, blood pressure, and myocardial oxygen demand. Adverse effects include bradycardia, hypotension, and atrioventricular conduction disturbances)

A school-age child presents with fever, myalgias, severe headache, photophobia, nausea, vomiting, and anorexia. On physical examination, an erythematous maculopapular rash is noted on the wrists and ankles. The MOST likely diagnosis is: A. Epstein-Barr virus (EBV) B. Lyme disease C. Meningococcemia D. Rocky Mountain spotted fever (RMSF)

D. Rocky Mountain spotted fever (RMSF) RMSF is a systemic, small-vessel vasculitis caused by Rickettsiaceae, a Gram-negative bacillus, which is transmitted to humans by ticks. The diagnosis is primarily clinical. Treatment includes antibiotics such as doxycycline. Like RMSF, Lyme disease is transmitted via ticks but is caused by the spirochete Borrelia burgdorferi. Manifestations can vary widely; however, early disease often includes a distinctive rash, erythema migrans. Rash begins as a red macule or papule that expands over days to weeks to form a large annual, erythematous lesion, sometimes with partial clearing leading to its target-like appearance. Constitutional symptoms may be present such as malaise, headache, mild neck stiffness, myalgia, and arthralgia. Meningococcemia is a life-threatening infection usually resulting in septicemia and/or meningitis. This presents with fever, chills, malaise, myalgia, limb pain, and a petechial rash. Progression can be rapid, leading to coa

Following resection of a medulloblastoma, a school-age child is noted as having irritability, a weight gain of 1.5 kg in the absence of edema, urine output of 0.4 mL/kg/hour, and the following laboratory results: Na+ 128, K+ 4.0, Cl- 103, Bicarb 26, BUN 10, Creatinine 0.2, Glucose 102, Ca+ 7.8, Mag 1.8, Phos 2.9 The MOST likely diagnosis is: A. Adrenal insufficiency B. Cerebral salt wasting C. Diabetes insipidus D. Syndrome of inappropriate antidiuretic hormone (SIADH)

D. Syndrome of inappropriate antidiuretic hormone (SIADH) Characterized by hyponatremia & decreased UOP due to excessive release of ADH. In SIADH the UOP is low (<1ml/kg/hr), leading to dilutional hyponatremia; whereas in CSW, UOP is elevated (2-3ml/kg/hr). Electrolyte abnormalities associated with CSW include an elevated BUN and hyponatremia due to renal losses of sodium. Children with SIADH have serum hypo-osmolarity. With diabetes insipidus (DI), there is a deficiency of ADH release, leading to the excretion of large amounts of dilute urine. An increased UOP of more than 4ml/kg/hr is seen & results in hypernatremia, increased serum osmolarity, and low urine osmolarity with a low urine specific gravity (< 1.005).

A child presents with fever, fatigue, arthralgias, and a photosensitive malar rash. Initial laboratory results include WBC 2,800, Hgb 11.7, Hct 34%, Plt 94,000, UA (pH 6.0, protein 4+, blood 1+, negative nitrites, ketones & leukocyte esterase). Which of the following is the MOST likely diagnosis? A. Henoch Schönlein purpura (HSP) B. Juvenile idiopathic arthritis (JIA) C. Kawasaki disease D. Systemic lupus erythematosus (SLE)

D. Systemic lupus erythematosus (SLE) Systemic lupus erythematosus (SLE): most commonly involved organ systems include the integumentary, musculoskeletal, renal, hematopoietic, vascular, & CNS. Diagnosis based on clinical & laboratory findings. The classic presentation includes rash, fever, and arthritis, the hallmark dermatologic finding is the malar, or butterfly, rash that is photosensitive. Renal manifestations include a proliferative glomerulonephritis. Hematologic findings include leukopenia (WBC <4,000), lymphopenia, and thrombocytopenia (plt <100,000). Elevated ANA commonly present in active SLE. Kawasaki disease: presents with acute onset of fever and rash. Diagnostic criteria include fever of a minimum of 5 days, conjunctival injection, rash, oral mucosal erythema with strawberry tongue and cracked lips, erythema and edema of hands and feet, rash, and lymphadenopathy (FIREY CRASH). Juvenile idiopathic arthritis (JIA): presents with fever, rash & various manifestations of ar

A child is resuscitated on location after a presumed submersion injury. Upon arrival the child is alert and fully conscious, and airway, perfusion, and respiratory effort are normal. Management includes supplemental oxygen, blood gas measurements, along with A. chest radiograph and IV antibiotics. B. IV diuretics and admission to general pediatric floor. C. IV steroids and admission to PICU. D. observation for at least 6 hours and discharge home if stable.

D. observation for at least 6 hours and discharge home if stable. Even those who have spontaneous ventilation and minimal or no neurologic dysfunction after resuscitation should be given supplemental oxygen to minimize the risk of progressive hypoxemia and acidosis with subsequent myocardial and cerebral damage.Physical examination has limited sensitivity for detecting hypoxemia. If oxygenation is confirmed as normal by pulse oximetry or blood gas determination while breathing room air, the likelihood of aspiration is minimal. Observation for a minimum of 6 hours with continued or repeat pulse oximetry or repeat blood gas determination should be sufficient to assess the possibility of late deterioration in gas exchange. When significant problems occur in gas exchange they typically present within 4 to 6 hours of submersion. Children who are observed for at least 6 hours and remain alert and fully conscious, may be discharged from the emergency department. The risk of pulmonary infect

An 8 year old with a traumatic grade IV liver laceration has an acute increase in work of breathing and decreased oxygen saturations requiring escalation of respiratory support 4 hours after completion of a packed red blood cell transfusion. A chest radiograph is obtained and reveals new bilateral infiltrates. Vital signs, respiratory support, and pertinent lab results Immediately PRIOR to transfusion include: HR 124, RR 20, O2 95% on 3 L NC, BP 80/56, CVP 4, hcb 6.9 & hct 21%. 4 hours AFTER tra

D. Transfusion-related acute lung injury (TRALI) TRALI presents with sudden onset of hypoxia and respiratory insufficiency during or shortly after blood product transfusion. It occurs most frequently during the transfusion but symptom onset can be delayed by as long as 6 hours following transfusion. Diagnosis depends on the following criteria: new acute respiratory distress syndrome occurring during or within 6 hours after blood product administration, documented hypoxemia, and bilateral pulmonary infiltrates on chest radiograph. Transfusion-associated circulatory overload (TACO) is a serious complication of blood transfusion that is common in adults and becoming more recognized in children. Although no single standard diagnostic criteria have been established for children, available descriptions include acute onset of respiratory distress within 6 hours following blood product transfusion, pulmonary edema identified on chest radiograph, and evidence of circulatory overload. Parameter

Within 2-3 days of initiating parenteral nutrition, which of the following laboratory assessments should be monitored to direct adjustment of the formulation? A. Complete blood count B. Prealbumin C. Trace elements D. Triglycerides

D. Triglycerides The following levels should be monitored and adjusted in the PN formulation as indicated: electrolytes and glucose should be checked daily until stable, BUN, creatinine, magnesium, calcium, and phosphorus should be checked twice a week, albumin, prealbumin, hepatic function, and CBC should be monitored weekly, triglycerides should be checked each time fat content is increased in the parenteral nutrition Triglyceride level can increase rapidly in the first several days after initiation of parenteral nutrition, therefore, a level should be checked 48-72 hours after initiation and the formulation of parenteral nutrition should be adjusted if the level is too high. Prealbumin, height, and weight are all important indicators of adequate nutritional status when on parenteral nutrition; however, changes in these parameters is slower to occur and would not warrant immediate change in formulation 2-3 days after initiation.

In the management of diabetic ketoacidosis (DKA), which of the following criteria demonstrates readiness for transition from a continuous insulin infusion to subcutaneous insulin therapy? A. Anion gap less than 25 mEq/L B. Bicarbonate greater than 12 mmol/L C. Serum glucose less than 250 mg/dl D. Venous pH greater than 7.30

D. Venous pH greater than 7.30 DKA occurs when severe insulin deficiency results in the inability of glucose to enter the cells, creating a starvation state. Initial management begins with fluid replacement. Insulin is administered via continuous intravenous infusion, which is closely titrated to reduce serum glucose no more than 100 mg/dL/hr. When serum glucose is below 250 mg/dL, dextrose is added to the intravenous fluids to provide a substrate for the insulin until normal glucose metabolism is restored and acidosis is corrected. Insulin administration by continuous IV infusion is maintained until the venous pH is greater than 7.30, serum ketones have cleared, and the anion gap has closed. A normal anion gap is 8 to 16 mEq/L. Once these goals have been achieved, insulin administration is transitioned to the subcutaneous route.

Which of the following has been shown to be the MOST sensitive indicator for early identification of compartment syndrome in children? A. pallor B. agitation C. absence of pulse D. increasing analgesic requirement

D. increasing analgesic requirement The most common precipitating event is trauma with associated bleeding into a muscle. Perfusion to the tissues decreases as the pressure rises, and resulting ischemia leads to cell death if not relieved. The traditional evaluation for compartment syndrome includes the 5 Ps of the neurovascular assessment:• pain,• paresthesia,• paralysis,• pallor, and• pulselessness. In pediatric populations, an increasing analgesia requirement should be considered an early indicator of compartment syndrome. The 5 Ps remain relevant for older children and adolescents who are able to report symptoms more accurately; thus, it is important to be cognizant of both the 5 Ps and the 3 As when assessing for indications of compartment syndrome in children.

A child with bacterial meningitis has a urine output that increases to 8 mL/kg/hour. Lab results include serum Na+ 150, serum osmolarity 310, & urine specific gravity 1.001. The MOST appropriate intervention is: A. fluid restriction B. hydrocortisone replacement C. intranasal desmopressin D. intravenous vasopressin

D. intravenous vasopressin The classic presentation of DI includes polyuria with a urine specific gravity of less than 1.005 in the presence of increased serum sodium and osmolarity. Critically ill children present with signs of dehydration, including dry mucous membranes, tachycardia, delayed capillary refill, cool extremities, and hypotension. The goals of therapy include correcting hemodynamic alterations, replacing the free water deficit, correcting hypernatremia, and restoring urine output to the normal range. The primary intervention is replacement of antidiuretic hormone. Intravenous vasopressin is the most appropriate preparation for use in critical care settings. Its short plasma half-life allows for rapid titration to achieve therapeutic goals. Desmopressin (DDAVP) is available in intranasal and oral formulations but has a much longer half-life and is not appropriate in the critical management of this child.

Which of the following represents information obtained during a "review of systems" (ROS)? A. height and weight B. maternal history of asthma C. number of previous hospitalizations D. presence of birthmarks or skin lesions

D. presence of birthmarks or skin lesions The ROS section of history-taking for a patient encounter incorporates specific subjective history about recent signs or symptoms by body system, as asked by the provider. The lead-in question is often "Has your child had any recent [fever, runny nose, rash, vomiting, etc.]." Family history would include information about parental health and medical problems, while height and weight would fall under objective data gathering. The number of previous hospitalizations is included in the chart or electronic medical record.

The MOST common cause of secondary hypertension in school-age children is: A. coarctation of the aorta B. obstructive sleep apnea C. pheochromocytomas D. renal parenchymal disease

D. renal parenchymal disease Acute and chronic renal parenchymal disease is the most common cause of secondary hypertension in children. Secondary hypertension should be suspected with younger age and a with more severely elevated blood pressure. Pediatric hypertension is defined as blood pressure measurements greater than the 95th percentile for age, gender, and height on three separate occasions. Essential (primary) hypertension has many causes, and children who are obese are more likely to develop essential hypertension. While coarctation of the aorta can present in older children, it typically presents during infancy. Catecholamine-secreting tumors such as a pheochromocytoma are an example of an endocrine related cause and can precipitate paroxysmal changes in blood pressure.

In preparation for a procedure, a neonate receives an intravenous dose of fentanyl and immediately responds with decreased oxygen saturations and cannot be mechanically ventilated. Administration of which of the following medications is the IMMEDIATE PRIORITY? A. ketamine B. midazolam C. propofol D. rocuronium

D. rocuronium Fentanyl is a synthetic opioid that functions as a selective agonist at the mu receptor sites to produce analgesia. It is quite potent and has a rapid onset and short duration of action compared to morphine, the gold standard opioid analgesic. Its rapid onset and short half-life lend well to its use for analgesia for procedural pain associated with brief procedures such as endotracheal intubation. In addition to adverse effects shared by other opioids, fentanyl has an infrequent, unique, and severe adverse effect of chest wall rigidity. This has been associated with higher individual (one-time or intermittent) doses and with rapid intravenous administration, although there are case reports of its occurrence in other situations. In this event, muscular rigidity of the chest wall interferes with both spontaneous and mechanically-administered breaths. Resolution requires pharmacologic rescue with either naloxone or a neuromuscular blocking agent, such as rocuronium or vec

A child presents with a limp. In confirming the diagnosis of septic arthritis as opposed to transient synovitis, which finding is MOST helpful? A. presence of a limp B. limited joint mobility C. normal sedimentation rate (ESR) D. temperature greater than 38.5° C (101.3° F)

D. temperature greater than 38.5° C (101.3° F) Children with septic arthritis often have a history of a recent upper respiratory or a local soft tissue infection for which they have received antibiotics. Common physical findings associated with septic arthritis include fever and limp, as well as erythema, warmth, tenderness, and swelling of the affected joint. If the hip is involved, the child may have referred pain to the anterior hip, groin, or thigh. The white blood cell count and differential, ESR, and C-reactive protein are generally elevated in children with joint infections, but are nonspecific and may not differentiate between infection and other inflammatory processes. Transient synovitis presents similarly, but without fever, and occurs most often in boys from 3 to 10 years of age. In this condition, the ESR and WBC are most often normal. Slipped capital femoral epiphysis (SCFE) is a non-inflammatory condition in which the femoral head is displaced. It is commonly seen i

A 5 year old recently diagnosed with neuroblastoma is currently receiving outpatient chemotherapy. Development of which of the following symptoms would indicate a medical emergency? A. anorexia B. hip pain C. mucosal ulcerations D. urinary retention

D. urinary retention Neuroblastoma is a tumor arising from cells in the sympathetic ganglia and adrenal medulla. It is the most commonly diagnosed malignancy in infancy, and the most common extracranial solid tumor in children. Children with neuroblastoma may present with a wide variety of findings, which depend to a large extent on the location of the tumor. The most common symptoms are due to the tumor mass or to bone pain from metastases. General symptoms may include failure to thrive, anorexia and fatigue. A persistent cough can arise from a thoracic mass. Older children who have neuroblastoma often have infiltration of the bone marrow with tumor causing pancytopenia. Bone involvement may produce pain with or without palpable bone masses. While any changes to bowel or bladder function are a concern for neurologic sequalae, retention of bowel or bladder contents can be significant for spinal cord compression by the tumor and is a medical emergency.

A child who sustained multiple traumas and is being mechanically ventilated develops an increasing oxygen requirement with new bilateral infiltrates on chest radiograph. Which of the following strategies is appropriate for management of conventional mechanical ventilation for this child? A. maintenance of oxygen saturations greater than 95% B. maintenance of PaCO2 in the low-normal range (35-40 mmHg) C. use of lower positive-end expiratory pressures (PEEP) D. use of lower tidal volumes

D. use of lower tidal volumes Acute respiratory distress syndrome (ARDS) can occur secondary to direct or indirect lung injury. ARDS is characterized by acute onset, a PaO2/FiO2 ratio of <200 mmHg, bilateral infiltrates on chest radiograph. Lung protective strategies utilized in management of ARDS include use of lower tidal volumes of 6-8 mL/kg on conventional mechanical ventilation and permissive hypercapnia. Use of lower tidal volumes minimizes further lung injury. Positive end-expiratory pressure (PEEP) has been shown to reverse hypoxemia associated with ARDS. A PEEP of 8 to 12 cm H2O or higher is commonly required. As PEEP increases, careful attention must be given to its cardiovascular effects. Oxygenation goals are for PaO2 55-80 mmHg or saturations of 88-95%.

An infant presents after an episode of choking during a feeding that was followed by cyanosis and a brief period of hypotonia. Previous evaluation following a similar episode one month ago revealed laryngomalacia and reflux for which a proton-pump inhibitor was prescribed. On physical examination, the infant is alert with mild stertorous breath sounds. A video fluoroscopic swallowing study demonstrates intermittent tracheal narrowing and a posterior, pulsatile indentation of the esophagus. Which

D. vascular ring Vascular rings are congenital structural or positional abnormalities of the aortic arch itself and/or its branches. The malformed vessels encircle and may impinge upon the trachea and/or esophagus. Presenting signs and symptoms vary among the different anatomic variations of vascular rings and the severity of compression of the trachea and/or esophagus. In infants, common symptoms include stridor, wheeze, or noisy breathing, and coughing, choking, and potential color change associated with feeding. For many infants, the first test that leads to identification of the final diagnosis of a vascular ring is either a bronchoscopy or a barium esophagogram obtained during assessment of respiratory symptoms or swallowing difficulty. Depending on the type of vascular ring and its impact on anatomy, indentations or filling defects in the esophagus and trachea may be identified. Pulsatility of the affected area identified on dynamic study is a key finding leading to final diagn


Kaugnay na mga set ng pag-aaral

Section 10: Unit 1: Introduction to Contracts and Michigan Contract Law

View Set

ATI ASSESSMENT B MED/SURG II (YAMILE)

View Set

Praxis 2 - Art Content Knowledge - Art Styles, Periods, and Specific Art, Printmaking, Photography, Color Theory, Ceramics - Study Guide Summer 2017

View Set

Unit 4 Week 5 - "Hey Nilda" & "Hi Rachel"

View Set

Cambridge Advanced Use of English Part 5 - Key word transformation (Quizlet Live)

View Set

Training and Development: CHAPTER 6

View Set

Principles of Management CH. 3&4 Cate Loes

View Set

Electromagnetic waves chapter 12

View Set